Board logo

标题: GMAT_RC_阅读总结 [打印本页]

作者: s    时间: 2010-11-22 21:02     标题: GMAT_RC_阅读总结

本帖最后由 s 于 2010-11-23 10:38 编辑

下载回复可见:[attach]9758[/attach]
目    录
GMAT阅读复习方法        4
一、总结        4
二、单词        4
三、难句        4
四、狗狗        4
GMAT阅读基础        4
一、题材类型        4
1.        自然科学        4
2.        社会科学        4
3.        经济管理        4
二、结构类型        4
1.        结构类型的区分        4
2.        新老观点型        5
3.        现象解释型        5
4.        问题解决型        5
5.        结论说明型        5
6.        比较分析型        5
三、作者态度        5
1.        正评价        5
2.        负评价        5
3.        混合评价:大正小负        5
4.        混合评价:大负小正        5
四、语言现象        5
1.        强转折        5
2.        强因果        6
3.        强对比        6
4.        强调词        6
5.        专有名词        6
6.        三种标点符号        6
7.        判断句        6
GMAT阅读方法        7
一、最高原则        7
二、阅读任务        7
1.        找出结构类型        7
2.        找出主题句        7
3.        找出主体词        7
4.        找出主体评价        7
5.        找出代表人物        7
三、笔记内容        7
1.        主题句        7
2.        主体及评价        7
3.        强对比        7
4.        强转折        7
四、快速阅读        7
1.        难句分析        7
2.        合理化取舍        8
3.        推测        8
五、出题点        9
GMAT解题方法        9
一、解题技巧        9
1.        阅读题目        9
2.        原文定位        10
3.        同义变换        10
二、客观题        10
1.        事实题        10
2.        取非题        10
3.        举例题        10
4.        列举题        11
5.        逻辑题        11
6.        信息题        11
7.        类比题        11
三、主观题        12
1.        主题题        12
2.        态度题        12
OG12阅读理解        14
一、阅读理解试题        14
Passage 1        14
Passage 2        15
Passage 3        16
Passage 4        18
Passage 5        19
Passage 6        22
Passage 7        23
Passage 8        25
Passage 9        26
Passage 10        28
Passage 11        29
Passage 12        31
Passage 13        33
Passage 14        35
Passage 15        37
Passage 16        39
Passage 17        41
Passage 18        42
Passage 19        44
Passage 20        46
Passage 21        48
Passage 22        50
Passage 23        52
Passage 24        54
二、阅读理解答案        56
GWD阅读理解        56
一、阅读理解试题        56
GWD-TN-14 Passage 1        56
GWD-TN-14 Passage 2        57
GWD-TN-14 Passage 3        58
GWD-TN-14 Passage 4        60
GWD-TN-15 Passage 1        61
GWD-TN-15 Passage 2        62
GWD-TN-15 Passage 3        64
GWD-TN-15 Passage 4        65
GWD-TN-16 Passage 1        66
GWD-TN-16 Passage 2        67
GWD-TN-16 Passage 3        69
GWD-TN-16 Passage 4        70
GWD-TN-17 Passage 1        71
GWD-TN-17 Passage 2        72
GWD-TN-17 Passage 3        74
GWD-TN-17 Passage 4        75
作者: s    时间: 2010-11-22 21:03

GMAT阅读复习方法
一、总结
   
二、单词
建立自己的阅读单词本,以动词为主。
三、难句
    记录和整理阅读难句
四、狗狗
    自己列提纲,剔除残狗,两遍以上,记住结构、问题和答案。
作者: s    时间: 2010-11-22 21:03

GMAT阅读基础
一、题材类型
1.        自然科学
重点是生命科学。注意区分fact和stuff,以及它们之间的逻辑。
2.        社会科学
    重点是美国历史,弱势群体(女性、黑人、少数民族)。注意列举。
3.        经济管理
    宏观与微观。注意推理,以及支持和反对。
二、结构类型
1.        结构类型的区分
  不要光看开头,看后面如何展开
  现象解释型:为什么?
  问题解决型:如何改变?
  结论解释型:有什么内容?
2.        新老观点型
老:1)喜新厌旧:traditionally / once / old / recently / until recently / 老时间
      2)标新立异:most / many / frequent(ly) / usually / common
  新:强转折however,新观点易在第一段中部或第二段开头出现
  主题句:明确代表新观点的那句话
3.        现象解释型
    现象:自然科学:自然现象
社会科学:史实(人物、时间、事件)
  解释:原因,主体,评价(即为什么发生?如何发生?):往往有多个理论,其写法、主体、评价、代表人物。
  主题句:正确的解释。(也可能没有,那就“看过程不看结果”。)
4.        问题解决型
    问题:疑问、任务
  疑问标志:疑问词how或what / question / puzzle / problem / 问号
  任务标志:task / difficulty / challenge / problem / criteria / requirement
  解决:疑问—回答;任务—完成,对多个方案的主体词和评价的注意
  主题句:正确的方案/答案
5.        结论说明型
结论:判断句
  解释:有什么内容,具体展开
主题句:开头的结论
6.        比较分析型
    对多个观点或方案进行比较,找出优缺点,他们之间的关系是平行关系,比如经济发展的编年史。不一定有单一主题句。
作者: s    时间: 2010-11-22 21:04

三、作者态度
1.        正评价
2.        负评价
3.        混合评价:大正小负
4.        混合评价:大负小正
四、语言现象
1.        强转折
虽然:(al)though(强调后者), while
  但是:but, yet, whereas, however
        in fact                        --        actually
        nevertheless                --        nonetheless
        rather                        --        instead
        despite                        --        in spite of
  注意:
      极端转折(最后半句是想说的)
      多重转折(粉笔是白的,可是有些粉笔是黑的;不过大多数情况下粉笔是白的,但不要忘了有变黑的可能性哦!——强调黑)
2.        强因果
表因为:because (of) / since / for / 冒号
    表所以:thus / hence / therefore / lead to / result in / result from / lie in
  表结论:conclude / conclusion / conclusive
  表后果:consequent(ly) / consequence
3.        强对比
    1)传统对比词:A. unlike(相反) B. contrast C. on the other hand D. on the contrary
    2)时间状语:A. before 1975 B. prior to C. in the 1950’s D. recently E. until recently
    取非题: Unlike B, A is ...,出题:B is ?
4.        强调词
    1)最高级
        普通最高级:most / least / -est
        顺序最高级:first, last
        频率最高级:always / never
        程度最高级:foremost / uttermost / utmost
    2)唯一性
        only / sole(ly) / unique(ly) / exclusive(ly) / alone(后置)
    3)比较级
        more than / less than(与其说…不如说…)/ as…as… / similar to / the same as
  慎重对待题干中的最高级、比较级
5.        专有名词
    人名、物种(动植物)、化学物质、地名,做首字母提取。
6.        三种标点符号
    1)引号
引用某人原话
表强调
表负评价
    2)括号、破折号
表插入、补充、解释
7.        判断句
定性结论、提纲挈领
1)谓语动词
    2)系动词、情态动词
        be / remain / seem / can / cannot / may / might
    3)自由褒贬词
                his view changed our attitude
                his view successfully(正)changed our attitude
                his curiously(负)changed our attitude(curiously:奇怪,居然)
                his penetrating(正)view changed our attitude
        his brazen(负)view changed our attitude
作者: s    时间: 2010-11-22 21:05

GMAT阅读方法
一、最高原则
主动阅读(快读文章慢做题)、原文定位(做笔记)、同义变换。
阅读水平提高的阶段:
    第一阶段:靠记忆
    第二阶段:靠定位和笔记
    第三阶段:靠结构
二、阅读任务
1.        找出结构类型
找出每个段落或层次,作者在写法上做了什么工作,包括:提出现象、提出解释、提出一个方法、提出一个问题、驳斥一个理论、验证一个理论。
2.        找出主题句
    根据结构类型找出文章的主题句。
3.        找出主体词
找出每个段落或层次的主体词、关键词是什么。
4.        找出主体评价
找出作者对每个主体有什么评价(与态度题密切相关)。
5.        找出代表人物
    找出每个段落或层次中主体行为的代表人物。
三、笔记内容
A4幅面,上下分开,标上段号1,2,3
1.        主题句
如:2L10ts(topic sentence)表示第2段第10行有主题句
2.        主体及评价
记录每段/层次的主体词及评价
3.        强对比
如:1L5C(Contrast)表示第1段第5行有强转折Contrast
4.        强转折
    如:1L5H(However)表示第1段第5行有强转折However
作者: s    时间: 2010-11-22 21:05

四、快速阅读
1.        难句分析
    结构:主干(主谓宾、主系表) / 修饰成分(大句子倒着分析,先找修饰成分)
    词:adj. adv.
    短语(介词短语):in the 1970’s
    从句(定语从句、状语从句)
  ☆ 把修饰成分用括号括起来,括不进去的就是主干
☆        从句结束的位置:
◇谓语动词/非谓语动词(现在分词、过去分词、不定式)
      ◇从句在无列举(无AND)的情况下,一般只能有一个谓语动词,所以从句在其开始的第一个和第二个谓语动词之间结束。
2.        合理化取舍
1)论点、论据、论证
    ★论点:
尽量看懂。尤其是表原因、方式的:by… / through / via / by doing
专业性文章:找主体词
    ★论据(举例、列举、数据、史实)
        ☆举例:
            紧跟结论的例子可以不看(如不放心,只看例子是关于谁的)
            篇幅在5行之内(机考就是7-8行之内,不足半屏)的例子可以不看
            出现在多个段落中的“例子”必须看(这种情况较少)
        ☆列举:
            小列举:列四项的小列举看,记住其位置
            大列举:
                找出标志词:first, second, in addition, finally
                找出列举中的主体词
        ☆数据;一个数字必须看,多个数字连续出现可以不看(因其十分明显,易于定位)
        ☆史实:人物、时间、事件(找动词) 、地点
    ★论证(找关系)
    ★因果:弄清论点和论据,哪几行是因,哪几行是果;是前因后果,还是前果后因。
    ★并列或递进:发生在多个论据之间
furthermore, moreover, in addition to
        first,一个原因second,一个原因
        1…, 2…, however(第三个原因最重要)…
2)不知道该不该看时,试探性地阅读,找整个句子的主语、谓语
3)整个文章的处理顺序
    文章结构=>重点细节(主体词)=>做题(查细节)
    阅读时间:做题时间 = 1:2或者2:3
4)不能舍掉的东西:
    ★ 强转折、强对比
    ★ “古怪”的语言:文章后半部分出现的新名词、临近结尾的转折、英文成语、修辞方法(主要是比喻)、不正常的“复合词”(作者生造出来的用 HYPHEN连接的词)
    ★ surprisingly / unfortunately / paradoxically / ironically开头的句子
3.        推测
  遇到单词或句子不理解时,从前后就近之处寻找相同题材的其他说法,如没有强转折、强对比,则它们的意思应该大体相同,正负评价应该一致;如有强转折、强对比,……
五、出题点
1.主题:主题题
包括内容性、写法性
  有明确主题句的文章出内容性主题题;
  套路清晰的文章出写法性主题题;
  套路特别清晰出organization题。
2.混和评价:态度题
3.强对比:取非题
4.强因果:事实题、改善题
  直接事实题(直接就原因或结果提问)
  改善题(由于某个原因导致某个理论“不好”)——回文找、取非
5.比较级:事实题、取非题
  正着出:直接事实题
  反着出:取非题
6.举例:举例题
举例作用题in order to
7.列举:列举题
记位置
8.延伸性内容:事实题
转为出题而设置
  直接事实题
  从前不从后原则(主题题中不能选延伸性内容的同意变换)
9.强调性语言和古怪的语言
最高级、唯一性、比较级、引号、括号、破折号
10.人物
尤其是引用其原话的(老观点中的不用看)
11.罗嗦内容
出现三四次
12.普遍性原理
某句话的适用范围超过了文章的适用范围,这句话就是普遍性原理。(as we all know)
作者: s    时间: 2010-11-22 21:06

GMAT解题方法
一、解题技巧
1.        阅读题目
1)分清四种错误选项
偏:以偏概全
混:张冠李戴,A说成B
反:与原文相反
无:原文没说或无法判断
2)慎重对待含有最高级、唯一性、比较级的选项
3)不要用非出题段落的内容来解答这个问题
    特殊:问每段开头时可以用上段结尾的内容来做题,反之亦然。
4)长选项竖读法
纵向比较所有选项,尤其是开头。找到其中相同的语言部分。如果某几个选项中的相同语言对这个题有用,先看这几个选项;否则先看其他选项,或先看这几个选项的差异部分。
5) 二选一时,找出区别,重新看问题,选答案
2.        原文定位
1)基本操作:主体词和评价
  文章:主体词┓
  题干:主体词┛
  回文中找相应语言就是原文定位。
2)特殊情况
  1)如果题干主体在文中多处出现,先看其首次出现之处
  2)若题干主体与文章主题有关,那么定位在主题句。
3.        同义变换
所谓“同义变换”,就是把文中的内容换成同义词或者变换句型,而意思保持不变。比如,文章说“fail to link”,到了题目里,改成“fail to relate”就是正确答案。再比如,“synthesize”改成“connect”,“broaden”改成“expand”,复杂一点的“lack access to”改成“not had sufficient opportunity to acquire”等。
二、客观题
1.        事实题
对文中事实正面提问
  ①定位(问的是哪一块东西,建立在读文章时对每段的基本总结,如主体词等,的基础上)
  ②同义变换(换词、换句式)
2.        取非题
1)对比题(由强对比出题)
2)改善题(虚拟+改善)
    用虚拟语气问一个事物或理论怎样会变得更好→回文中找缺点,然后取非(针对不好的才会出改善题)
    better / improve / more illuminating / more enlightening / more convincing
    文:犯罪率是不好研究的,因为总人口不确定。
    问:怎样才能搞好犯罪率的研究?
    答:弄清总人口。
3)改善题的变体
    回文中找优点,然后取非。
3.        举例题
  不是例子题有in order to也当例子题做。
  窍门:先看开头为give an example of / illustrate的选项
  标准做法:
      1)找例子的结论
          ☆for example / instance, such as, like (先结论后举例)
          ☆因果关系词(先举例后结论)
          ☆无语言现象,先按第一种做
      2)同义变换
4.        列举题
1)小列举(文中用逗号隔开的)
    列出四项会出“排除题”EXCEPT…(回文中定位)
    不是四项的不用看。
2)大列举(FIRST, SECOND,…)
    “罗马数字题”(回原文定位)
    特殊情况:有些罗马数字题与大列举无关,是信息题。
5.        逻辑题
1)支持题
support / strengthen
  A.搭桥法:把缺的条件补上去
  B.罗嗦法:直接重复结论
  C.具体事例法:举文章没说的例子作正面论据
2)反对题
    weaken / undermine / call into question / cast doubt on
  A.釜底抽薪:反对原因
  B.欲擒故纵:反证法
      用虚拟语气承认B对,推出一个矛盾、谬误或不可能的情况,结论:B不成立
  C.抬杠法:反对结论
  D.他因法:存在其他原因法
      反对A推出B,证C推出B即可。
3)例题
题干:唐人街40%的集资合同是用C方式集资的,说明中国人最喜欢C
    釜底抽薪:1.研究表明唐人街95%以上的集资合同都是朝鲜人和越南人签的
              2.有53%是A方式的
    欲擒故纵:如果中国人最喜欢C的话,那么今天唐人街就不会有任何一家中餐馆了
    抬杠法:研究表明中国人就是不喜欢C集资方式
    他因法:美国法律规定唐人街必须有40%的合同是用C方式集资的
    搭桥法:唐人街95%以上的集资合同是中国人签的,而且没有其他集资方式超过C
    啰嗦法:研究表明中国人的确最喜欢C集资方式
    具体事例法:纽约、芝加哥和洛杉矶三大城市的中国人最喜欢C方式
6.        信息题
  问法:
      1) which of the following is (not) true?
      2)文章的信息(没)回答下列哪个问题?
      3)作者(不)同意下列哪种说法?
  类型:
      1)有定位的of, about, concerning, regarding (关于)
      2)无定位的(花时间多、不一定能作对,可舍弃)
          根据对全文的整体把握,逐个判断选项。
7.        类比题
  问法:which of the following is most similar to the situation in…? (analogous to / parallel / consistent with)
  解法:抓本质特征
作者: s    时间: 2010-11-22 21:06

三、主观题
1.        主题题
1.问法:
main idea / primary idea / main purpose / thesis, mainly concerned with, organization, best title
2.分类:
内容性主题题:关键词、细节
写法性主题题:结构、写法动词,describe, explain, summarize, refute
  区分:从选项上
3.做法:
  内容性:
      1)如文中有明确主题句,对其进行同义变换
      2)无明确主题句,用“三出现原则”:
重要关键词(主体词)必须出现;
        细节内容不能出现;
新内容不能出现。
      3)主题题包含态度时,选出来的选项的态度应与文中保持一致
  写法性:
      1)一切要从套路出发来做
      2)特殊情况:
          A.organization:文章套路“特别清晰”(板块分明)时考
          B.特别套路:评述某人理论/著作:多数属于结论解释型,少数属于现象解释型。在做主题题的时候,直接选评述某人理论/著作这样的说法。在特别套路中,作者一般避免给全正或全负的评价,而是给混和评价。
4.二原则:
  1)从前不从后:
      “延伸性内容”:在文末或某段段末出现的与主体关系不密切并可能“升华”到更高高度的内容(“跑题”的内容,与主题无关)。如文章前后出现了不一样的重点、不一样的结论,那么以前面的为主题,后面的为延伸性内容。
  2)从大不从小:
      文章开头或主题句中的评价是大评价(让步和转折除外)
2.        态度题
1.判断依据:
  1)判断句(系动词、情态动词、自由褒贬词)
  2)某些引号表负评价
  3)某些实义动词表评价,尤其在研究类的文章中:
      A.负:fail to / overestimate / underestimate / exaggerate / misinterpret / misrepresent / ignore(主观) / neglect / overlook
      B.正:find / discover / show / demonstrate / prove / confirm / determine
  4)让步:前半句是假话;后半句是真正要说的,转折:一般大负小正。
A. it is true / correct / of course, 强转折
      B. no doubt / without doubt / undoubtedly, 强转折
      C.助动词do / may / seem / appear, 强转折
  5)从大不从小原则:从全文来讲,看前面、开头或主题讲的是大评价。
2.做题注意事项:
    1)顺序
        先在文中找到评价,再看选项,不要先看选项
  2)混和评价的正确选项的表达方式:
      A.…but / and…
        如:critical but admiring (大正小负)
        correct but limited
correct and incomplete
      B.由修饰词所限定的评价词:
          partially (correct); qualified (approval)(打了折扣的,有限制的);
          guarded (criticism) (慎重的);tentative (acceptance)(暂时的,尝试性的)
          reluctant (criticism)(不情愿的);enthusiasm tempered by minor reservation
    3)避免极端选项:
      A.对于讲义中所列的词,不要自己再增加新的。
      B.极端选项排除法只是个辅助的方法,适用范围较窄。
作者: s    时间: 2010-11-22 21:06

OG12阅读理解
一、阅读理解试题
Passage 1
题材类型:经济管理
结构类型:结论说明型
组织方式:总分,批评=>理由1=>理由2=>改进
主要对象:ecoefficiency
作者态度:±
Ecoefficiency (measures to minimize environmental impact through the reduction or elimination of waste from production processes) has become a goal for companies worldwide, with many realizing significant cost savings from such innovations. Peter Senge and Goran Carstedt see this development as laudable but suggest that simply adopting ecoefficiency innovations could actually worsen environmental stresses in the future. Such innovations reduce production waste but do not alter the number of products manufactured nor the waste generated from their use and discard; indeed, most companies invest in ecoefficiency improvements in order to increase profits and growth. Moreover, there is no guarantee that increased economic growth from ecoefficiency will come in similarly ecoefficient ways, since in today’s global markets, greater profits may be turned into investment capital that could easily be reinvested in old-style eco-inefficient industries. Even a vastly more ecoefficient industrial system could, were it to grow much larger, generate more total waste and destroy more habitat and species than would a smaller, less ecoefficient economy. Senge and Carstedt argue that to preserve the global environment and sustain economic growth, businesses must develop a new systemic approach that reduces total material use and total accumulated waste. Focusing exclusively on ecoefficiency, which offers a compelling business case according to established thinking, may distract companies from pursuing radically different products and business models.
Questions 1–3 refer to the passage above.
1. The primary purpose of the passage is to (主旨)
(A) explain why a particular business strategy has been less successful than was once anticipated
(B) propose an alternative to a particular business strategy that has inadvertently caused ecological damage
(C) present a concern about the possible consequences of pursuing a particular business strategy
(D) make a case for applying a particular business strategy on a larger scale than is currently practiced
(E) suggest several possible outcomes of companies’ failure to understand the economic impact of a particular business strategy

2. The passage mentions which of the following as a possible consequence of companies’ realization of greater profits through ecoefficiency? (细节)
(A) The companies may be able to sell a greater number of products by lowering prices.
(B) The companies may be better able to attract investment capital in the global market.
(C) The profits may be reinvested to increase economic growth through ecoefficiency.
(D) The profits may be used as investment capital for industries that are not ecoefficient.
(E) The profits may encourage companies to make further innovations in reducing production waste.

3. The passage implies that which of the following is a possible consequence of a company’s adoption of innovations that increase its ecoefficiency? (细节)
(A) Company profits resulting from such innovations may be reinvested in that company with no guarantee that the company will continue to make further improvements in ecoefficiency.
(B) Company growth fostered by cost savings from such innovations may allow that company to manufacture a greater number of products that will be used and discarded, thus worsening environmental stress.
(C) A company that fails to realize significant cost savings from such innovations may have little incentive to continue to minimize the environmental impact of its production processes.
(D) A company that comes to depend on such innovations to increase its profits and growth may be vulnerable in the global market to competition from old-style eco-inefficient industries.
(E) A company that meets its ecoefficiency goals is unlikely to invest its increased profits in the development of new and innovative ecoefficiency measures.
作者: s    时间: 2010-11-22 21:07

Passage 2
题材类型:自然科学
结构类型:现象解释型
组织方式:总分总,发现=>排除其他解释=>提出合理解释(结论)
主要对象:predator-prey
作者态度:±
A recent study has provided clues to predator-prey dynamics in the late Pleistocene era. Researchers compared the number of tooth fractures in present-day carnivores with tooth fractures in carnivores that lived 36,000 to 10,000 years ago and that were preserved in the Rancho La Brea tar pits in Los Angeles. The breakage frequencies in the extinct species were strikingly higher than those in the present-day species.
In considering possible explanations for this finding, the researchers dismissed demographic bias because older individuals were not overrepresented in the fossil samples. They rejected preservational bias because a total absence of breakage in two extinct species demonstrated that the fractures were not the result of abrasion within the pits. They ruled out local bias because breakage data obtained from other Pleistocene sites were similar to the La Brea data. The explanation they consider most plausible is behavioral differences between extinct and present-day carnivores—in particular, more contact between the teeth of predators and the bones of prey due to more thorough consumption of carcasses by the extinct species. Such thorough carcass consumption implies to the researchers either that prey availability was low, at least seasonally, or that there was intense competition over kills and a high rate of carcass theft due to relatively high predator densities.
Questions 4–8 refer to the passage above.
4. The primary purpose of the passage is to (主旨)
(A) present several explanations for a well-known fact
(B) suggest alternative methods for resolving a debate
(C) argue in favor of a controversial theory
(D) question the methodology used in a study
(E) discuss the implications of a research finding

5. According to the passage, compared with Pleistocene carnivores in other areas, Pleistocene carnivores in the La Brea area (细节)
(A) included the same species, in approximately the same proportions
(B) had a similar frequency of tooth fractures
(C) populated the La Brea area more densely
(D) consumed their prey more thoroughly
(E) found it harder to obtain sufficient prey

6. According to the passage, the researchers believe that the high frequency of tooth breakage in carnivores found at La Brea was caused primarily by (细节)
(A) the aging process in individual carnivores
(B) contact between the fossils in the pits
(C) poor preservation of the fossils after they were removed from the pits
(D) the impact of carnivores’ teeth against the bones of their prey
(E) the impact of carnivores’ teeth against the bones of other carnivores during fights over kills

7. The researchers’ conclusion concerning the absence of demographic bias would be most seriously undermined if it were found that (逻辑)
(A) the older an individual carnivore is, the more likely it is to have a large number of tooth fractures
(B) the average age at death of a present-day carnivore is greater than was the average age at death of a Pleistocene carnivore
(C) in Pleistocene carnivore species, older individuals consumed carcasses as thoroughly as did younger individuals
(D) the methods used to determine animals’ ages in fossil samples tend to misidentify many older individuals as younger individuals
(E) data concerning the ages of fossil samples cannot provide reliable information about behavioral differences between extinct carnivores and present-day carnivores

8. According to the passage, if the researchers had NOT found that two extinct carnivore species were free of tooth breakage, the researchers would have concluded that (推论)
(A) the difference in breakage frequencies could have been the result of damage to the fossil remains in the La Brea pits
(B) the fossils in other Pleistocene sites could have higher breakage frequencies than do the fossils in the La Brea pits
(C) Pleistocene carnivore species probably behaved very similarly to one another with respect to consumption of carcasses
(D) all Pleistocene carnivore species differed behaviorally from present-day carnivore species
(E) predator densities during the Pleistocene era were extremely high
作者: s    时间: 2010-11-22 21:07

Passage 3
题材类型:社会科学
结构类型:问题回答型
组织方式:总分,提出问题=>提出方案=>分析可行性
主要对象:archaeology
作者态度:±
Archaeology as a profession faces two major problems. First, it is the poorest of the poor. Only paltry sums are available for excavating and even less is available for publishing the results and preserving the sites once excavated. Yet archaeologists deal with priceless objects every day. Second, there is the problem of illegal excavation, resulting in museum-quality pieces being sold to the highest bidder.
I would like to make an outrageous suggestion that would at one stroke provide funds for archaeology and reduce the amount of illegal digging. I would propose that scientific archaeological expeditions and governmental authorities sell excavated artifacts on the open market. Such sales would provide substantial funds for the excavation and preservation of archaeological sites and the publication of results. At the same time, they would break the illegal excavator’s grip on the market, thereby decreasing the inducement to engage in illegal activities.
You might object that professionals excavate to acquire knowledge, not money. Moreover, ancient artifacts are part of our global cultural heritage, which should be available for all to appreciate, not sold to the highest bidder. I agree. Sell nothing that has unique artistic merit or scientific value. But, you might reply, everything that comes out of the ground has scientific value. Here we part company. Theoretically, you may be correct in claiming that every artifact has potential scientific value. Practically, you are wrong.
I refer to the thousands of pottery vessels and ancient lamps that are essentially duplicates of one another. In one small excavation in Cyprus, archaeologists recently uncovered 2,000 virtually indistinguishable small jugs in a single courtyard. Even precious royal seal impressions known as l’melekh handles have been found in abundance —more than 4,000 examples so far.
The basements of museums are simply not large enough to store the artifacts that are likely to be discovered in the future. There is not enough money even to catalog the finds; as a result, they cannot be found again and become as inaccessible as if they had never been discovered. Indeed, with the help of a computer, sold artifacts could be more accessible than are the pieces stored in bulging museum basements. Prior to sale, each could be photographed and the list of the purchasers could be maintained on the computer. A purchaser could even be required to agree to return the piece if it should become needed for scientific purposes.
It would be unrealistic to suggest that illegal digging would stop if artifacts were sold on the open market. But the demand for the clandestine product would be substantially reduced. Who would want an unmarked pot when another was available whose provenance was known, and that was dated stratigraphically by the professional archaeologist who excavated it?
Questions 9–11 refer to the passage above.
9. The primary purpose of the passage is to propose (主旨)
(A) an alternative to museum display of artifacts
(B) a way to curb illegal digging while benefiting the archaeological profession
(C) a way to distinguish artifacts with scientific value from those that have no such value
(D) the governmental regulation of archaeological sites
(E) a new system for cataloging duplicate artifacts

10. The author implies that all of the following statements about duplicate artifacts are true EXCEPT (细节)
(A) a market for such artifacts already exists
(B) such artifacts seldom have scientific value
(C) there is likely to be a continuing supply of such artifacts
(D) museums are well supplied with examples of such artifacts
(E) such artifacts frequently exceed in quality those already cataloged in museum collections

11. Which of the following is mentioned in the passage as a disadvantage of storing artifacts in museum basements? (细节)
(A) Museum officials rarely allow scholars access to such artifacts.
(B) Space that could be better used for display is taken up for storage.
(C) Artifacts discovered in one excavation often become separated from each other.
(D) Such artifacts are often damaged by variations in temperature and humidity.
(E) Such artifacts often remain uncataloged and thus cannot be located once they are put in storage.
作者: s    时间: 2010-11-22 21:21

Passage 4
题材类型:经济管理
结构类型:新老观点型
组织方式:总分,旧理论=>新理论=>举例=>分析
主要对象:integrated approaches
作者态度:±
Traditionally, the first firm to commercialize a new technology has benefited from the unique opportunity to shape product definitions, forcing followers to adapt to a standard or invest in an unproven alternative. Today, however, the largest payoffs may go to companies that lead in developing integrated approaches for successful mass production and distribution.
Producers of the Beta format for videocassette recorders (VCRs), for example, were first to develop the VCR commercially in 1975, but producers of the rival VHS (Video Home System) format proved to be more successful at forming strategic alliances with other producers and distributors to manufacture and market their VCR format. Seeking to maintain exclusive control over VCR distribution, Beta producers were reluctant to form such alliances and eventually lost ground to VHS in the competition for the global VCR market.
Despite Beta’s substantial technological head start and the fact that VHS was neither technically better nor cheaper than Beta, developers of VHS quickly turned a slight early lead in sales into a dominant position. Strategic alignments with producers of prerecorded tapes reinforced the VHS advantage. The perception among consumers that prerecorded tapes were more available in VHS format further expanded VHS’s share of the market. By the end of the 1980s, Beta was no longer in production.
Questions 12–17 refer to the passage above.
12. The passage is primarily concerned with which of the following? (主旨)
(A) Evaluating two competing technologies
(B) Tracing the impact of a new technology by narrating a sequence of events
(C) Reinterpreting an event from contemporary business history
(D) Illustrating a business strategy by means of a case history
(E) Proposing an innovative approach to business planning

13. According to the passage, today’s successful firms, unlike successful firms in the past, may earn the greatest profits by (细节)
(A) investing in research to produce cheaper versions of existing technology
(B) being the first to market a competing technology
(C) adapting rapidly to a technological standard previously set by a competing firm
(D) establishing technological leadership in order to shape product definitions in advance of competing firms
(E) emphasizing the development of methods for the mass production and distribution of a new technology

14. According to the passage, consumers began to develop a preference for VCRs in the VHS format because they believed which of the following? (细节)
(A) VCRs in the VHS format were technically better than competing format VCRs.
(B) VCRs in the VHS format were less expensive than competing format VCRs.
(C) VHS was the first standard format for VCRs.
(D) VHS prerecorded videotapes were more available than those in Beta format.
(E) VCRs in the Beta format would soon cease to be produced.

15. The author implies that one way that VHS producers won control over the VCR market was by (细节)
(A) carefully restricting access to VCR technology
(B) giving up a slight early lead in VCR sales in order to improve long-term prospects
(C) retaining a strict monopoly on the production of prerecorded videotapes
(D) sharing control of the marketing of VHS format VCRs
(E) sacrificing technological superiority over Beta format VCRs in order to remain competitive in price

16. The alignment of producers of VHS format VCRs with producers of prerecorded videotapes is most similar to which of the following? (应用)
(A) The alignment of an automobile manufacturer with another automobile manufacturer to adopt a standard design for automobile engines
(B) The alignment of an automobile manufacturer with an automotive glass company whereby the manufacturer agrees to purchase automobile windshields only from that one glass company
(C) The alignment of an automobile manufacturer with a petroleum company to ensure the widespread availability of the fuel required by a new type of engine developed by the manufacturer
(D) The alignment of an automobile manufacturer with its dealers to adopt a plan to improve automobile design
(E) The alignment of an automobile dealer with an automobile rental chain to adopt a strategy for an advertising campaign to promote a new type of automobile

17. Which of the following best describes the relation of the first paragraph to the passage as a whole? (结构)
(A) It makes a general observation to be exemplified.
(B) It outlines a process to be analyzed.
(C) It poses a question to be answered.
(D) It advances an argument to be disputed.
(E) It introduces conflicting arguments to be reconciled.
作者: s    时间: 2010-11-22 21:22

Passage 5
题材类型:自然科学
结构类型:结论说明型
组织方式:总分,理论=>例子1=>例子2
主要对象:adaptations
作者态度:+
In terrestrial environments, gravity places special demands on the cardiovascular systems of animals. Gravitational pressure can cause blood to pool in the lower regions of the body, making it difficult to circulate blood to critical organs such as the brain. Terrestrial snakes, in particular, exhibit adaptations that aid in circulating blood against the force of gravity.
The problem confronting terrestrial snakes is best illustrated by what happens to sea snakes when removed from their supportive medium. Because the vertical pressure gradients within the blood vessels are counteracted by similar pressure gradients in the surrounding water, the distribution of blood throughout the body of sea snakes remains about the same regardless of their orientation in space, provided they remain in the ocean. When removed from the water and tilted at various angles with the head up, however, blood pressure at their midpoint drops significantly, and at brain level falls to zero. That many terrestrial snakes in similar spatial orientations do not experience this kind of circulatory failure suggests that certain adaptations enable them to regulate blood pressure more effectively in those orientations.
One such adaptation is the closer proximity of the terrestrial snake’s heart to its head, which helps to ensure circulation to the brain, regardless of the snake’s orientation in space. The heart of sea snakes can be located near the middle of the body, a position that minimizes the work entailed in circulating blood to both extremities. In arboreal snakes, however, which dwell in trees and often assume a vertical posture, the average distance from the heart to the head can be as little as 15 percent of overall body length. Such a location requires that blood circulated to the tail of the snake travel a greater distance back to the heart, a problem solved by another adaptation. When climbing, arboreal snakes often pause momentarily to wiggle their bodies, causing waves of muscle contraction that advance from the lower torso to the head. By compressing the veins and forcing blood forward, these contractions apparently improve the flow of venous blood returning to the heart.
Questions 18–25 refer to the passage above.
18. The passage provides information in support of which of the following assertions? (应用)
(A) The disadvantages of an adaptation to a particular feature of an environment often outweigh the advantages of such an adaptation.
(B) An organism’s reaction to being placed in an environment to which it is not well adapted can sometimes illustrate the problems that have been solved by the adaptations of organisms indigenous to that environment.
(C) The effectiveness of an organism’s adaptation to a particular feature of its environment can only be evaluated by examining the effectiveness with which organisms of other species have adapted to a similar feature of a different environment.
(D) Organisms of the same species that inhabit strikingly different environments will often adapt in remarkably similar ways to the few features of those environments that are common.
(E) Different species of organisms living in the same environment will seldom adapt to features of that environment in the same way.

19. According to the passage, one reason that the distribution of blood in the sea snake changes little while the creature remains in the ocean is that (细节)
(A) the heart of the sea snake tends to be located near the center of its body
(B) pressure gradients in the water surrounding the sea snake counter the effects of vertical pressure gradients within its blood vessels
(C) the sea snake assumes a vertical posture less frequently than do the terrestrial and the arboreal snake
(D) the sea snake often relies on waves of muscle contractions to help move blood from the torso to the head
(E) the force of pressure gradients in the water surrounding the sea snake exceeds that of vertical pressure gradients within its circulatory system

20. It can be inferred from the passage that which of the following is true of species of terrestrial snakes that often need to assume a vertical posture? (推论)
(A) They are more likely to be susceptible to circulatory failure in vertical postures than are sea snakes.
(B) Their hearts are less likely to be located at the midpoint of their bodies than is the case with sea snakes.
(C) They cannot counteract the pooling of blood in lower regions of their bodies as effectively as sea snakes can.
(D) The blood pressure at their midpoint decreases significantly when they are tilted with their heads up.
(E) They are unable to rely on muscle contractions to move venous blood from the lower torso to the head.

21. The author describes the behavior of the circulatory system of sea snakes when they are removed from the ocean (see lines 17–20) primarily in order to (评价)
(A) illustrate what would occur in the circulatory system of terrestrial snakes without adaptations that enable them to regulate their blood pressure in vertical orientations
(B) explain why arboreal snakes in vertical orientations must rely on muscle contractions to restore blood pressure to the brain
(C) illustrate the effects of circulatory failure on the behavior of arboreal snakes
(D) illustrate the superiority of the circulatory system of the terrestrial snake to that of the sea snake
(E) explain how changes in spatial orientation can adversely affect the circulatory system of snakes with hearts located in relatively close proximity to their heads

22. It can be inferred from the passage that which of the following is a true statement about sea snakes? (推论)
(A) They frequently rely on waves of muscle contractions from the lower torso to the head to supplement the work of the heart.
(B) They cannot effectively regulate their blood pressure when placed in seawater and tilted at an angle with the head pointed downward.
(C) They are more likely to have a heart located in close proximity to their heads than are arboreal snakes.
(D) They become acutely vulnerable to the effects of gravitational pressure on their circulatory system when they are placed in a terrestrial environment.
(E) Their cardiovascular system is not as complicated as that of arboreal snakes.

23. The author suggests that which of the following is a disadvantage that results from the location of a snake’s heart in close proximity to its head? (细节)
(A) A decrease in the efficiency with which the snake regulates the flow of blood to the brain
(B) A decrease in the number of orientations in space that a snake can assume without loss of blood flow to the brain
(C) A decrease in blood pressure at the snake’s midpoint when it is tilted at various angles with its head up
(D) An increase in the tendency of blood to pool at the snake’s head when the snake is tilted at various angles with its head down
(E) An increase in the amount of effort required to distribute blood to and from the snake’s tail

24. The primary purpose of the third paragraph is to (评价)
(A) introduce a topic that is not discussed earlier in the passage
(B) describe a more efficient method of achieving an effect discussed in the previous paragraph
(C) draw a conclusion based on information elaborated in the previous paragraph
(D) discuss two specific examples of phenomena mentioned at the end of the previous paragraph
(E) introduce evidence that undermines a view reported earlier in the passage

25. In the passage, the author is primarily concerned with doing which of the following? (主旨)
(A) Explaining adaptations that enable the terrestrial snake to cope with the effects of gravitational pressure on its circulatory system
(B) Comparing the circulatory system of the sea snake with that of the terrestrial snake
(C) Explaining why the circulatory system of the terrestrial snake is different from that of the sea snake
(D) Pointing out features of the terrestrial snake’s cardiovascular system that make it superior to that of the sea snake
(E) Explaining how the sea snake is able to neutralize the effects of gravitational pressure on its circulatory system
作者: s    时间: 2010-11-22 21:22

Passage 6
题材类型:经济管理
结构类型:比较分析型
组织方式:平行,政策1=>政策2=>政策3
主要对象:economic development strategy
作者态度:±
During the 1960s and 1970s, the primary economic development strategy of local governments in the United States was to attract manufacturing industries. Unfortunately, this strategy was usually implemented at another community’s expense: many manufacturing facilities were lured away from their moorings elsewhere through tax incentives and slick promotional efforts. Through the transfer of jobs and related revenues that resulted from this practice, one town’s triumph could become another town’s tragedy.
In the 1980s the strategy shifted from this zero-sum game to one called “high-technology development,” in which local governments competed to attract newly formed high-technology manufacturing firms. Although this approach was preferable to victimizing other geographical areas by taking their jobs, it also had its shortcomings: high-tech manufacturing firms employ only a specially trained fraction of the manufacturing workforce, and there simply are not enough high-tech firms to satisfy all geographic areas.
Recently, local governments have increasingly come to recognize the advantages of yet a third strategy: the promotion of homegrown small businesses. Small indigenous businesses are created by a nearly ubiquitous resource, local entrepreneurs. With roots in their communities, these individuals are less likely to be enticed away by incentives offered by another community. Indigenous industry and talent are kept at home, creating an environment that both provides jobs and fosters further entrepreneurship.
Questions 26–30 refer to the passage above.
26. The primary purpose of the passage is to (主旨)
(A) advocate more effective strategies for encouraging the development of hightechnology enterprises in the United States
(B) contrast the incentives for economic development offered by local governments with those offered by the private sector
(C) acknowledge and counter adverse criticism of programs being used to stimulate local economic development
(D) define and explore promotional efforts used by local governments to attract new industry
(E) review and evaluate strategies and programs that have been used to stimulate economic development

27. The passage suggests which of the following about the majority of United States manufacturing industries before the high-technology development era of the 1980s? (细节)
(A) They lost many of their most innovative personnel to small entrepreneurial enterprises.
(B) They experienced a major decline in profits during the 1960s and 1970s.
(C) They could provide real economic benefits to the areas in which they were located.
(D) They employed workers who had no specialized skills.
(E) They actively interfered with local entrepreneurial ventures.

28. The tone of the passage suggests that the author is most optimistic about the economic development potential of which of the following groups? (语气)
(A) Local governments
(B) High-technology promoters
(C) Local entrepreneurs
(D) Manufacturing industry managers
(E) Economic development strategists

29. The passage does NOT state which of the following about local entrepreneurs? (细节)
(A) They are found nearly everywhere.
(B) They encourage further entrepreneurship.
(C) They attract out-of-town investors.
(D) They employ local workers.
(E) They are established in their communities.

30. The author of the passage mentions which of the following as an advantage of high-technology development? (细节)
(A) It encourages the modernization of existing manufacturing facilities.
(B) It promotes healthy competition between rival industries.
(C) It encourages the growth of related industries.
(D) It takes full advantage of the existing workforce.
(E) It does not advantage one local workforce at the expense of another.
作者: s    时间: 2010-11-22 21:22

Passage 7
题材类型:经济管理
结构类型:比较分析型
组织方式:平行,定义1=>分析1=>定义2=>分析2
主要对象:service worker
作者态度:±
In 1988 services moved ahead of manufacturing as the main product of the United States economy. But what is meant by “services”? Some economists define a service as something that is produced and consumed simultaneously, for example, a haircut. The broader, classical definition is that a service is an intangible something that cannot be touched or stored. Yet electric utilities can store energy, and computer programmers save information electronically. Thus, the classical definition is hard to sustain.
The United States government’s definition is more practical: services are the residual category that includes everything that is not agriculture or industry. Under this definition, services includes activities as diverse as engineering and driving a bus. However, besides lacking a strong conceptual framework, this definition fails to recognize the distinction between service industries and service occupations. It categorizes workers based on their company’s final product rather than on the actual work the employees perform. Thus, the many service workers employed by manufacturers— bookkeepers or janitors, for example—would fall under the industrial rather than the services category. Such ambiguities reveal the arbitrariness of this definition and suggest that, although practical for government purposes, it does not accurately reflect the composition of the current United States economy.
Questions 31–35 refer to the passage above.
31. The author of the passage is primarily concerned with (主旨)
(A) discussing research data underlying several definitions
(B) arguing for the adoption of a particular definition
(C) exploring definitions of a concept
(D) comparing the advantages of several definitions
(E) clarifying some ambiguous definitions

32. In comparing the United States government’s definition of services with the classical definition, the author suggests that the classical definition is (推论)
(A) more pragmatic
(B) more difficult to apply
(C) less ambiguous
(D) more widely used
(E) more arbitrary

33. The passage suggests which of the following about service workers in the United States? (推论)
(A) The number of service workers may be underestimated by the definition of services used by the government.
(B) There were fewer service workers than agricultural workers before 1988.
(C) The number of service workers was almost equal to the number of workers employed in manufacturing until 1988.
(D) Most service workers are employed in service occupations rather than in service industries.
(E) Most service workers are employed in occupations where they provide services that do not fall under the classical definition of services.

34. The author of the passage mentions which of the following as one disadvantage of the United States government’s definition of services? (细节)
(A) It is less useful than the other definitions mentioned in the passage.
(B) It is narrower in scope than the other definitions mentioned in the passage.
(C) It is based on the final product produced rather than on the type of work performed.
(D) It does not recognize the diversity of occupations within the service industries.
(E) It misclassifies many workers who are employed in service industries.

35. The author refers to “service workers employed by manufacturers” (line 23) primarily in order to point out (结构)
(A) a type of worker not covered by the United States government’s system of classifying occupations
(B) a flaw in the United States government’s definition of services
(C) a factor that has influenced the growth of the service economy in the United States
(D) a type of worker who is classified on the basis of work performed rather than on the basis of the company’s final product
(E) the diversity of the workers who are referred to as service workers
作者: s    时间: 2010-11-22 21:23

Passage 8
题材类型:社会科学
结构类型:新老观点型
组织方式:总分,研究方法=>优点=>缺点=>例子
主要对象:oral narratives
作者态度:±
Current feminist theory, in validating women’s own stories of their experience, has encouraged scholars of women’s history to view the use of women’s oral narratives as the methodology, next to the use of women’s written autobiography, that brings historians closest to the “reality” of women’s lives. Such narratives, unlike most standard histories, represent experience from the perspective of women, affirm the importance of women’s contributions, and furnish present-day women with historical continuity that is essential to their identity, individually and collectively.
Scholars of women’s history should, however, be as cautious about accepting oral narratives at face value as they already are about written memories. Oral narratives are no more likely than are written narratives to provide a disinterested commentary on events or people. Moreover, the stories people tell to explain themselves are shaped by narrative devices and storytelling conventions, as well as by other cultural and historical factors, in ways that the storytellers may be unaware of. The political rhetoric of a particular era, for example, may influence women’s interpretations of the significance of their experience. Thus a woman who views the Second World War as pivotal in increasing the social acceptance of women’s paid work outside the home may reach that conclusion partly and unwittingly because of wartime rhetoric encouraging a positive view of women’s participation in such work.
Questions 36–41 refer to the passage above.
36. The passage is primarily concerned with (主旨)
(A) contrasting the benefits of one methodology with the benefits of another
(B) describing the historical origins and inherent drawbacks of a particular methodology
(C) discussing the appeal of a particular methodology and some concerns about its use
(D) showing that some historians’ adoption of a particular methodology has led to criticism of recent historical scholarship
(E) analyzing the influence of current feminist views on women’s interpretations of their experience

37. According to the passage, which of the following shapes the oral narratives of women storytellers? (细节)
(A) The conventions for standard histories in the culture in which a woman storyteller lives
(B) The conventions of storytelling in the culture in which a woman storyteller lives
(C) A woman storyteller’s experience with distinctive traditions of storytelling developed by the women in her family of origin
(D) The cultural expectations and experiences of those who listen to oral narratives
(E) A woman storyteller’s familiarity with the stories that members of other groups in her culture tell to explain themselves

38. The author of the passage would be most likely to make which of the following recommendations to scholars of women’s history? (应用)
(A) They should take into account their own life experiences when interpreting the oral accounts of women’s historical experiences.
(B) They should assume that the observations made in women’s oral narratives are believed by the intended audience of the story.
(C) They should treat skeptically observations reported in oral narratives unless the observations can be confirmed in standard histories.
(D) They should consider the cultural and historical context in which an oral narrative was created before arriving at an interpretation of such a narrative.
(E) They should rely on information gathered from oral narratives only when equivalent information is not available in standard histories.

39. Which of the following best describes the function of the last sentence of the passage? (评价)
(A) It describes an event that historians view as crucial in recent women’s history.
(B) It provides an example of how political rhetoric may influence the interpretations of experience reported in women’s oral narratives.
(C) It provides an example of an oral narrative that inaccurately describes women’s experience during a particular historical period.
(D) It illustrates the point that some women are more aware than others of the social forces that shape their oral narratives.
(E) It identifies the historical conditions that led to the social acceptance of women’s paid work outside the home.

40. According to the passage, scholars of women’s history should refrain from doing which of the following? (推论)
(A) Relying on traditional historical sources when women’s oral narratives are unavailable
(B) Focusing on the influence of political rhetoric on women’s perceptions to the exclusion of other equally important factors
(C) Attempting to discover the cultural and historical factors that influence the stories women tell
(D) Assuming that the conventions of women’s written autobiographies are similar to the conventions of women’s oral narratives
(E) Accepting women’s oral narratives less critically than they accept women’s written histories

41. According to the passage, each of the following is a difference between women’s oral narratives and most standard histories EXCEPT: (细节)
(A) Women’s oral histories validate the significance of women’s achievements.
(B) Women’s oral histories depict experience from the point of view of women.
(C) Women’s oral histories acknowledge the influence of well-known women.
(D) Women’s oral histories present today’s women with a sense of their historical relationship to women of the past.
(E) Women’s oral histories are crucial to the collective identity of today’s women.
作者: s    时间: 2010-11-22 21:23

Passage 9
题材类型:社会科学
结构类型:问题回答型
组织方式:总分,提出问题=>分析问题=>提出方案=>优点
主要对象:texts
作者态度:+
In recent years, teachers of introductory courses in Asian American studies have been facing a dilemma nonexistent a few decades ago, when hardly any texts in that field were available. Today, excellent anthologies and other introductory texts exist, and books on individual Asian American nationality groups and on general issues important for Asian Americans are published almost weekly. Even professors who are experts in the field find it difficult to decide which of these to assign to students; nonexperts who teach in related areas and are looking for writings for and by Asian Americans to include in survey courses are in an even worse position.
A complicating factor has been the continuing lack of specialized one-volume reference works on Asian Americans, such as biographical dictionaries or desktop encyclopedias. Such works would enable students taking Asian American studies courses (and professors in related fields) to look up basic information on Asian American individuals, institutions, history, and culture without having to wade through mountains of primary source material. In addition, given such works, Asian American studies professors might feel more free to include more challenging Asian American material in their introductory reading lists, since good reference works allow students to acquire on their own the background information necessary to interpret difficult or unfamiliar material.
Questions 42–46 refer to the passage above.
42. The author of the passage is primarily concerned with doing which of the following? (主旨)
(A) Recommending a methodology
(B) Describing a course of study
(C) Discussing a problem
(D) Evaluating a past course of action
(E) Responding to a criticism

43. The “dilemma” mentioned in line 3 can best be characterized as being caused by the necessity to make a choice when faced with a (推论)
(A) lack of acceptable alternatives
(B) lack of strict standards for evaluating alternatives
(C) preponderance of bad alternatives as compared to good
(D) multitude of different alternatives
(E) large number of alternatives that are nearly identical in content

44. The passage suggests that the factor mentioned in lines 15–18 complicates professors’ attempts to construct introductory reading lists for courses in Asian American studies in which of the following ways? (推论)
(A) By making it difficult for professors to identify primary source material and to obtain standard information on Asian American history and culture
(B) By preventing professors from identifying excellent anthologies and introductory texts in the field that are both recent and understandable to students
(C) By preventing professors from adequately evaluating the quality of the numerous texts currently being published in the field
(D) By making it more necessary for professors to select readings for their courses that are not too challenging for students unfamiliar with Asian American history and culture
(E) By making it more likely that the readings professors assign to students in their courses will be drawn solely from primary sources

45. The passage implies that which of the following was true of introductory courses in Asian American studies a few decades ago? (推论)
(A) The range of different textbooks that could be assigned for such courses was extremely limited.
(B) The texts assigned as readings in such courses were often not very challenging for students.
(C) Students often complained about the texts assigned to them in such courses.
(D) Such courses were offered only at schools whose libraries were rich in primary sources.
(E) Such courses were the only means then available by which people in the United States could acquire knowledge of the field.

46. According to the passage, the existence of good one-volume reference works about Asian Americans could result in (细节)
(A) increased agreement among professors of Asian American studies regarding the quality of the sources available in their field
(B) an increase in the number of students signing up for introductory courses in Asian American studies
(C) increased accuracy in writings that concern Asian American history and culture
(D) the use of introductory texts about Asian American history and culture in courses outside the field of Asian American studies
(E) the inclusion of a wider range of Asian American material in introductory reading lists in Asian American studies
作者: s    时间: 2010-11-22 21:23

Passage 10
题材类型:经济理论
结构类型:现象解释型
组织方式:总分,现象=>经典理论=>解释=>不足=>补充
主要对象:segregation of labor
作者态度:±
In the seventeenth-century Florentine textile industry, women were employed primarily in low-paying, low-skill jobs. To explain this segregation of labor by gender, economists have relied on the useful theory of human capital. According to this theory, investment in human capital—the acquisition of difficult job-related skills—generally benefits individuals by making them eligible to engage in well-paid occupations. Women’s role as child bearers, however, results in interruptions in their participation in the job market (as compared with men’s) and thus reduces their opportunities to acquire training for highly skilled work. In addition, the human capital theory explains why there was a high concentration of women workers in certain low-skill jobs, such as weaving, but not in others, such as combing or carding, by positing that because of their primary responsibility in child rearing women took occupations that could be carried out in the home.
There were, however, differences in pay scales that cannot be explained by the human capital theory. For example, male construction workers were paid significantly higher wages than female taffeta weavers. The wage difference between these two low-skill occupations stems from the segregation of labor by gender: because a limited number of occupations were open to women, there was a large supply of workers in their fields, and this “overcrowding” resulted in women receiving lower wages and men receiving higher wages.
Questions 47–49 refer to the passage above.
47. The passage suggests that combing and carding differ from weaving in that combing and carding were (细节)
(A) low-skill jobs performed primarily by women employees
(B) low-skill jobs that were not performed in the home
(C) low-skill jobs performed by both male and female employees
(D) high-skill jobs performed outside the home
(E) high-skill jobs performed by both male and female employees

48. Which of the following, if true, would most weaken the explanation provided by the human capital theory for women’s concentration in certain occupations in seventeenth-century Florence? (逻辑)
(A) Women were unlikely to work outside the home even in occupations whose hours were flexible enough to allow women to accommodate domestic tasks as well as paid labor.
(B) Parents were less likely to teach occupational skills to their daughters than they were to their sons.
(C) Women’s participation in the Florentine paid labor force grew steadily throughout the sixteenth and seventeenth centuries.
(D) The vast majority of female weavers in the Florentine wool industry had children.
(E) Few women worked as weavers in the Florentine silk industry, which was devoted to making cloths that required a high degree of skill to produce.

49. The author of the passage would be most likely to describe the explanation provided by the human capital theory for the high concentration of women in certain occupations in the seventeenth-century Florentine textile industry as (逻辑)
(A) well founded though incomplete
(B) difficult to articulate
(C) plausible but poorly substantiated
(D) seriously flawed
(E) contrary to recent research
作者: s    时间: 2010-11-22 21:24

Passage 11  
题材类型:经济理论
结构类型:新老观点型
组织方式:总分,旧观点=>新观点=>解释=>建议
主要对象:temporary employment
作者态度:±
(This passage was adapted from an article written in 1992.)
Some observers have attributed the dramatic growth in temporary employment that occurred in the United States during the 1980s to increased participation in the workforce by certain groups, such as first-time or reentering workers, who supposedly prefer such arrangements. However, statistical analyses reveal that demographic changes in the workforce did not correlate with variations in the total number of temporary workers. Instead, these analyses suggest that factors affecting employers account for the rise in temporary employment. One factor is product demand: temporary employment is favored by employers who are adapting to fluctuating demand for products while at the same time seeking to reduce overall labor costs. Another factor is labor’s reduced bargaining strength, which allows employers more control over the terms of employment. Given the analyses, which reveal that growth in temporary employment now far exceeds the level explainable by recent workforce entry rates of groups said to prefer temporary jobs, firms should be discouraged from creating excessive numbers of temporary positions. Government policymakers should consider mandating benefit coverage for temporary employees, promoting pay equity between temporary and permanent workers, assisting labor unions in organizing temporary workers, and encouraging firms to assign temporary jobs primarily to employees who explicitly indicate that preference.
Questions 50–56 refer to the passage above.
50. The primary purpose of the passage is to (主旨)
(A) present the results of statistical analyses and propose further studies
(B) explain a recent development and predict its eventual consequences
(C) identify the reasons for a trend and recommend measures to address it
(D) outline several theories about a phenomenon and advocate one of them
(E) describe the potential consequences of implementing a new policy and argue in favor of that policy

51. According to the passage, which of the following is true of the “factors affecting employers” that are mentioned in lines 10–19? (细节)
(A) Most experts cite them as having initiated the growth in temporary employment that occurred during the 1980s.
(B) They may account for the increase in the total number of temporary workers during the 1980s.
(C) They were less important than demographic change in accounting for the increase of temporary employment during the 1980s.
(D) They included a sharp increase in the cost of labor during the 1980s.
(E) They are more difficult to account for than are other factors involved in the growth of temporary employment during the 1980s.

52. The passage suggests which of the following about the use of temporary employment by firms during the 1980s? (推论)
(A) It enabled firms to deal with fluctuating product demand far more efficiently than they did before the 1980s.
(B) It increased as a result of increased participation in the workforce by certain demographic groups.
(C) It was discouraged by government-mandated policies.
(D) It was a response to preferences indicated by certain employees for more flexible working arrangements.
(E) It increased partly as a result of workers’ reduced ability to control the terms of their employment.

53. The passage suggests which of the following about the workers who took temporary jobs during the 1980s? (推论)
(A) Their jobs frequently led to permanent positions within firms.
(B) They constituted a less demographically diverse group than has been suggested.
(C) They were occasionally involved in actions organized by labor unions.
(D) Their pay declined during the decade in comparison with the pay of permanent employees.
(E) They did not necessarily prefer temporary employment to permanent employment.

54. The first sentence in the passage suggests that the “observers” mentioned in line 1 would be most likely to predict which of the following? (应用)
(A) That the number of new temporary positions would decline as fewer workers who preferred temporary employment entered the workforce
(B) That the total number of temporary positions would increase as fewer workers were able to find permanent positions
(C) That employers would have less control over the terms of workers’ employment as workers increased their bargaining strength
(D) That more workers would be hired for temporary positions as product demand increased
(E) That the number of workers taking temporary positions would increase as more workers in any given demographic group entered the workforce

55. In the context of the passage, the word “excessive” (line 23) most closely corresponds to which of the following phrases? (逻辑)
(A) Far more than can be justified by worker preferences
(B) Far more than can be explained by fluctuations in product demand
(C) Far more than can be beneficial to the success of the firms themselves
(D) Far more than can be accounted for by an expanding national economy
(E) Far more than can be attributed to increases in the total number of people in the workforce

56. The passage mentions each of the following as an appropriate kind of governmental action EXCEPT (细节)
(A) getting firms to offer temporary employment primarily to a certain group of people
(B) encouraging equitable pay for temporary and permanent employees
(C) facilitating the organization of temporary workers by labor unions
(D) establishing guidelines on the proportion of temporary workers that firms should employ
(E) ensuring that temporary workers obtain benefits from their employers
作者: s    时间: 2010-11-22 21:24

Passage 12
题材类型:社会科学
结构类型:比较分析型
组织方式:平行,法律1=>分析1=>法律2=>分析2
主要对象:legal bases for the water rights
作者态度:±
In Winters v. United States (1908), the Supreme Court held that the right to use waters flowing through or adjacent to the Fort Belknap Indian Reservation was reserved to American Indians by the treaty establishing the reservation. Although this treaty did not mention water rights, the Court ruled that the federal government, when it created the reservation, intended to deal fairly with American Indians by reserving for them the waters without which their lands would have been useless. Later decisions, citing Winters, established that courts can find federal rights to reserve water for particular purposes if (1) the land in question lies within an enclave under exclusive federal jurisdiction, (2) the land has been formally withdrawn from federal public lands—i.e., withdrawn from the stock of federal lands available for private use under federal land use laws—and set aside or reserved, and (3) the circumstances reveal the government intended to reserve water as well as land when establishing the reservation.
Some American Indian tribes have also established water rights through the courts based on their traditional diversion and use of certain waters prior to the United States’ acquisition of sovereignty. For example, the Rio Grande pueblos already existed when the United States acquired sovereignty over New Mexico in 1848. Although they at that time became part of the United States, the pueblo lands never formally constituted a part of federal public lands; in any event, no treaty, statute, or executive order has ever designated or withdrawn the pueblos from public lands as American Indian reservations. This fact, however, has not barred application of the Winters doctrine. What constitutes an American Indian reservation is a question of practice, not of legal definition, and the pueblos have always been treated as reservations by the United States. This pragmatic approach is buttressed by Arizona v. California (1963), wherein the Supreme Court indicated that the manner in which any type of federal reservation is created does not affect the application to it of the Winters doctrine. Therefore, the reserved water rights of Pueblo Indians have priority over other citizens’ water rights as of 1848, the year in which pueblos must be considered to have become reservations.
Questions 57–63 refer to the passage above.
57. According to the passage, which of the following was true of the treaty establishing the Fort Belknap Indian Reservation? (细节)
(A) It was challenged in the Supreme Court a number of times.
(B) It was rescinded by the federal government, an action that gave rise to the Winters case.
(C) It cited American Indians’ traditional use of the land’s resources.
(D) It failed to mention water rights to be enjoyed by the reservation’s inhabitants.
(E) It was modified by the Supreme Court in Arizona v. California.

58. The passage suggests that, if the criteria discussed in lines 10–20 were the only criteria for establishing a reservation’s water rights, which of the following would be true? (推论)
(A) The water rights of the inhabitants of the Fort Belknap Indian Reservation would not take precedence over those of other citizens.
(B) Reservations established before 1848 would be judged to have no water rights.
(C) There would be no legal basis for the water rights of the Rio Grande pueblos.
(D) Reservations other than American Indian reservations could not be created with reserved water rights.
(E) Treaties establishing reservations would have to mention water rights explicitly in order to reserve water for a particular purpose.

59. Which of the following most accurately summarizes the relationship between Arizona v. California in lines 38–42, and the criteria citing the Winters doctrine in lines 10–20? (推论)
(A) Arizona v. California abolishes these criteria and establishes a competing set of criteria for applying the Winters doctrine.
(B) Arizona v. California establishes that the Winters doctrine applies to a broader range of situations than those defined by these criteria.
(C) Arizona v. California represents the sole example of an exception to the criteria as they were set forth in the Winters doctrine.
(D) Arizona v. California does not refer to the Winters doctrine to justify water rights, whereas these criteria do rely on the Winters doctrine.
(E) Arizona v. California applies the criteria derived from the Winters doctrine only to federal lands other than American Indian reservations.

60. The “pragmatic approach” mentioned in lines 37–38 of the passage is best defined as one that (细节)
(A) grants recognition to reservations that were never formally established but that have traditionally been treated as such
(B) determines the water rights of all citizens in a particular region by examining the actual history of water usage in that region
(C) gives federal courts the right to reserve water along with land even when it is clear that the government originally intended to reserve only the land
(D) bases the decision to recognize the legal rights of a group on the practical effect such a recognition is likely to have on other citizens
(E) dictates that courts ignore precedents set by such cases as Winters v. United States in deciding what water rights belong to reserved land

61. The author cites the fact that the Rio Grande pueblos were never formally withdrawn from public lands primarily in order to do which of the following? (评价)
(A) Suggest why it might have been argued that the Winters doctrine ought not to apply to pueblo lands
(B) Imply that the United States never really acquired sovereignty over pueblo lands
(C) Argue that the pueblo lands ought still to be considered part of federal public lands
(D) Support the argument that the water rights of citizens other than American Indians are limited by the Winters doctrine
(E) Suggest that federal courts cannot claim jurisdiction over cases disputing the traditional diversion and use of water by Pueblo Indians

62. The primary purpose of the passage is to (主旨)
(A) trace the development of laws establishing American Indian reservations
(B) explain the legal bases for the water rights of American Indian tribes
(C) question the legal criteria often used to determine the water rights of American Indian tribes
(D) discuss evidence establishing the earliest date at which the federal government recognized the water rights of American Indians
(E) point out a legal distinction between different types of American Indian reservations

63. The passage suggests that the legal rights of citizens other than American Indians to the use of water fl owing into the Rio Grande pueblos are (推论)
(A) guaranteed by the precedent set in Arizona v. California
(B) abolished by the Winters doctrine
(C) deferred to the Pueblo Indians whenever treaties explicitly require this
(D) guaranteed by federal land-use laws
(E) limited by the prior claims of the Pueblo Indians
作者: s    时间: 2010-11-22 21:24

Passage 13
题材类型:经济管理
结构类型:结论说明型
组织方式:总分,制度=>结果=>分析=>举例
主要对象:import relief
作者态度:±
Many United States companies have, unfortunately, made the search for legal protection from import competition into a major line of work. Since 1980 the United States International Trade Commission (ITC) has received about 280 complaints alleging damage from imports that benefit from subsidies by foreign governments. Another 340 charge that foreign companies “dumped” their products in the United States at “less than fair value.” Even when no unfair practices are alleged, the simple claim that an industry has been injured by imports is sufficient grounds to seek relief.
Contrary to the general impression, this quest for import relief has hurt more companies than it has helped. As corporations begin to function globally, they develop an intricate web of marketing, production, and research relationships. The complexity of these relationships makes it unlikely that a system of import relief laws will meet the strategic needs of all the units under the same parent company.
Internationalization increases the danger that foreign companies will use import relief laws against the very companies the laws were designed to protect. Suppose a United States–owned company establishes an overseas plant to manufacture a product while its competitor makes the same product in the United States. If the competitor can prove injury from the imports—and that the United States company received a subsidy from a foreign government to build its plant abroad—the United States company’s products will be uncompetitive in the United States, since they would be subject to duties.
Perhaps the most brazen case occurred when the ITC investigated allegations that Canadian companies were injuring the United States salt industry by dumping rock salt, used to de-ice roads. The bizarre aspect of the complaint was that a foreign conglomerate with United States operations was crying for help against a United States company with foreign operations. The “United States” company claiming injury was a subsidiary of a Dutch conglomerate, while the “Canadian’’ companies included a subsidiary of a Chicago firm that was the second-largest domestic producer of rock salt.
Questions 64–69 refer to the passage above.
64. The passage is chiefly concerned with (主旨)
(A) arguing against the increased internationalization of United States corporations
(B) warning that the application of laws affecting trade frequently has unintended consequences
(C) demonstrating that foreign-based firms receive more subsidies from their governments than United States firms receive from the United States government
(D) advocating the use of trade restrictions for “dumped” products but not for other imports
(E) recommending a uniform method for handling claims of unfair trade practices

65. It can be inferred from the passage that the minimal basis for a complaint to the International Trade Commission is which of the following? (推论)
(A) A foreign competitor has received a subsidy from a foreign government.
(B) A foreign competitor has substantially increased the volume of products shipped to the United States.
(C) A foreign competitor is selling products in the United States at less than fair market value.
(D) The company requesting import relief has been injured by the sale of imports in the United States.
(E) The company requesting import relief has been barred from exporting products to the country of its foreign competitor.

66. The last paragraph performs which of the following functions in the passage? (结构)
(A) It summarizes the discussion thus far and suggests additional areas for research.
(B) It presents a recommendation based on the evidence presented earlier.
(C) It discusses an exceptional case in which the results expected by the author of the passage were not obtained.
(D) It introduces an additional area of concern not mentioned earlier.
(E) It cites a specific case that illustrates a problem presented more generally in the previous paragraph.

67. The passage warns of which of the following dangers? (细节)
(A) Companies in the United States may receive no protection from imports unless they actively seek protection from import competition.
(B) Companies that seek legal protection from import competition may incur legal costs that far exceed any possible gain.
(C) Companies that are United States owned but operate internationally may not be eligible for protection from import competition under the laws of the countries in which their plants operate.
(D) Companies that are not United States owned may seek legal protection from import competition under United States import relief laws.
(E) Companies in the United States that import raw materials may have to pay duties on those materials.

68. The passage suggests that which of the following is most likely to be true of United States trade laws? (推论)
(A) They will eliminate the practice of “dumping” products in the United States.
(B) They will enable manufacturers in the United States to compete more profitably outside the United States.
(C) They will affect United States trade with Canada more negatively than trade with other nations.
(D) Those that help one unit within a parent company will not necessarily help other units in the company.
(E) Those that are applied to international companies will accomplish their intended result.

69. It can be inferred from the passage that the author believes which of the following about the complaint mentioned in the last paragraph? (推论)
(A) The ITC acted unfairly toward the complainant in its investigation.
(B) The complaint violated the intent of import relief laws.
(C) The response of the ITC to the complaint provided suitable relief from unfair trade practices to the complainant.
(D) The ITC did not have access to appropriate information concerning the case.
(E) Each of the companies involved in the complaint acted in its own best interest.
作者: s    时间: 2010-11-22 21:24

Passage 14
题材类型:自然科学
结构类型:问题回答型
组织方式:总分总,提出问题=>解决方案=>原理=>优点=>补充
主要对象:chronology of the ice ages
作者态度:+
Milankovitch proposed in the early twentieth century that the ice ages were caused by variations in the Earth’s orbit around the Sun. For some time this theory was considered untestable, largely because there was no sufficiently precise chronology of the ice ages with which the orbital variations could be matched.
To establish such a chronology it is necessary to determine the relative amounts of land ice that existed at various times in the Earth’s past. A recent discovery makes such a determination possible: relative land-ice volume for a given period can be deduced from the ratio of two oxygen isotopes, 16 and 18, found in ocean sediments. Almost all the oxygen in water is oxygen 16, but a few molecules out of every thousand incorporate the heavier isotope 18. When an ice age begins, the continental ice sheets grow, steadily reducing the amount of water evaporated from the ocean that will eventually return to it. Because heavier isotopes tend to be left behind when water evaporates from the ocean surfaces, the remaining ocean water becomes progressively enriched in oxygen 18. The degree of enrichment can be determined by analyzing ocean sediments of the period, because these sediments are composed of calcium carbonate shells of marine organisms, shells that were constructed with oxygen atoms drawn from the surrounding ocean. The higher the ratio of oxygen 18 to oxygen 16 in a sedimentary specimen, the more land ice there was when the sediment was laid down.
As an indicator of shifts in the Earth’s climate, the isotope record has two advantages. First, it is a global record: there is remarkably little variation in isotope ratios in sedimentary specimens taken from different continental locations. Second, it is a more continuous record than that taken from rocks on land. Because of these advantages, sedimentary evidence can be dated with sufficient accuracy by radiometric methods to establish a precise chronology of the ice ages. The dated isotope record shows that the fluctuations in global ice volume over the past several hundred thousand years have a pattern: an ice age occurs roughly once every 100,000 years. These data have established a strong connection between variations in the Earth’s orbit and the periodicity of the ice ages.
However, it is important to note that other factors, such as volcanic particulates or variations in the amount of sunlight received by the Earth, could potentially have affected the climate. The advantage of the Milankovitch theory is that it is testable; changes in the Earth’s orbit can be calculated and dated by applying Newton’s laws of gravity to progressively earlier configurations of the bodies in the solar system. Yet the lack of information about other possible factors affecting global climate does not make them unimportant.
Questions 70–75 refer to the passage above.
70. In the passage, the author is primarily interested in (主旨)
(A) suggesting an alternative to an outdated research method
(B) introducing a new research method that calls an accepted theory into question
(C) emphasizing the instability of data gathered from the application of a new scientific method
(D) presenting a theory and describing a new method to test that theory
(E) initiating a debate about a widely accepted theory

71. The author of the passage would be most likely to agree with which of the following statements about the Milankovitch theory? (应用)
(A) It is the only possible explanation for the ice ages.
(B) It is too limited to provide a plausible explanation for the ice ages, despite recent research findings.
(C) It cannot be tested and confirmed until further research on volcanic activity is done.
(D) It is one plausible explanation, though not the only one, for the ice ages.
(E) It is not a plausible explanation for the ice ages, although it has opened up promising possibilities for future research.

72. It can be inferred from the passage that the isotope record taken from ocean sediments would be less useful to researchers if which of the following were true? (推论)
(A) It indicated that lighter isotopes of oxygen predominated at certain times.
(B) It had far more gaps in its sequence than the record taken from rocks on land.
(C) It indicated that climate shifts did not occur every 100,000 years.
(D) It indicated that the ratios of oxygen 16 and oxygen 18 in ocean water were not consistent with those found in fresh water.
(E) It stretched back for only a million years.

73. According to the passage, which of the following is true of the ratios of oxygen isotopes in ocean sediments? (细节)
(A) They indicate that sediments found during an ice age contain more calcium carbonate than sediments formed at other times.
(B) They are less reliable than the evidence from rocks on land in determining the volume of land ice.
(C) They can be used to deduce the relative volume of land ice that was present when the sediment was laid down.
(D) They are more unpredictable during an ice age than in other climatic conditions.
(E) They can be used to determine atmospheric conditions at various times in the past.

74. It can be inferred from the passage that precipitation formed from evaporated ocean water has (推论)
(A) the same isotopic ratio as ocean water
(B) less oxygen 18 than does ocean water
(C) less oxygen 18 than has the ice contained in continental ice sheets
(D) a different isotopic composition than has precipitation formed from water on land
(E) more oxygen 16 than has precipitation formed from fresh water

75. It can be inferred from the passage that calcium carbonate shells (推论)
(A) are not as susceptible to deterioration as rocks
(B) are less common in sediments formed during an ice age
(C) are found only in areas that were once covered by land ice
(D) contain radioactive material that can be used to determine a sediment’s isotopic composition
(E) reflect the isotopic composition of the water at the time the shells were formed
作者: s    时间: 2010-11-22 21:25

Passage 15
题材类型:社会科学
结构类型:比较分析型
组织方式:总分,两部著作=>两种方法=>对比分析
主要对象:women’s status
作者态度:±
Two works published in 1984 demonstrate contrasting approaches to writing the history of United States women. Buel and Buel’s biography of Mary Fish (1736–1818) makes little effort to place her story in the context of recent historiography on women. Lebsock, meanwhile, attempts not only to write the history of women in one southern community, but also to redirect two decades of historiographical debate as to whether women gained or lost status in the nineteenth century as compared with the eighteenth century. Although both books offer the reader the opportunity to assess this controversy regarding women’s status, only Lebsock’s deals with it directly. She examines several different aspects of women’s status, helping to refine and resolve the issues. She concludes that while women gained autonomy in some areas, especially in the private sphere, they lost it in many aspects of the economic sphere. More importantly, she shows that the debate itself depends on frame of reference: in many respects, women lost power in relation to men, for example, as certain jobs (delivering babies, supervising schools) were taken over by men. Yet women also gained power in comparison with their previous status, owning a higher proportion of real estate, for example. In contrast, Buel and Buel’s biography provides ample raw material for questioning the myth, fostered by some historians, of a colonial golden age in the eighteenth century but does not give the reader much guidance in analyzing the controversy over women’s status.
Questions 76–81 refer to the passage above.
76. The primary purpose of the passage is to (主旨)
(A) examine two sides of a historiographical debate
(B) call into question an author’s approach to a historiographical debate
(C) examine one author’s approach to a historiographical debate
(D) discuss two authors’ works in relationship to a historiographical debate
(E) explain the prevalent perspective on a historiographical debate

77. The author of the passage mentions the supervision of schools primarily in order to (评价)
(A) remind readers of the role education played in the cultural changes of the nineteenth century in the United States
(B) suggest an area in which nineteenth-century American women were relatively free to exercise power
(C) provide an example of an occupation for which accurate data about women’s participation are difficult to obtain
(D) speculate about which occupations were considered suitable for United States women of the nineteenth century
(E) illustrate how the answers to questions about women’s status depend on particular contexts

78. With which of the following characterizations of Lebsock’s contribution to the controversy concerning women’s status in the nineteenth-century United States would the author of the passage be most likely to agree? (细节)
(A) Lebsock has studied women from a formerly neglected region and time period.
(B) Lebsock has demonstrated the importance of frame of reference in answering questions about women’s status.
(C) Lebsock has addressed the controversy by using women’s current status as a frame of reference.
(D) Lebsock has analyzed statistics about occupations and property that were previously ignored.
(E) Lebsock has applied recent historiographical methods to the biography of a nineteenth-century woman.

79. According to the passage, Lebsock’s work differs from Buel and Buel’s work in that Lebsock’s work (细节)
(A) uses a large number of primary sources
(B) ignores issues of women’s legal status
(C) refuses to take a position on women’s status in the eighteenth century
(D) addresses larger historiographical issues
(E) fails to provide sufficient material to support its claims

80. The passage suggests that Lebsock believes that compared to nineteenth-century American women, eighteenth-century American women were (推论)
(A) in many respects less powerful in relation to men
(B) more likely to own real estate
(C) generally more economically independent
(D) more independent in conducting their private lives
(E) less likely to work as school superintendents

81. The passage suggests that Buel and Buel’s biography of Mary Fish provides evidence for which of the following views of women’s history? (推论)
(A) Women have lost power in relation to men since the colonial era.
(B) Women of the colonial era were not as likely to be concerned with their status as were women in the nineteenth century.
(C) The colonial era was not as favorable for women as some historians have believed.
(D) Women had more economic autonomy in the colonial era than in the nineteenth century.
(E) Women’s occupations were generally more respected in the colonial era than in the nineteenth century.
作者: s    时间: 2010-11-22 21:25

Passage 16
题材类型:自然科学
结构类型:现象解释型
组织方式:总分,发现=>实验1=>实验2=>解释=>实验3=>解释
主要对象:serotonin
作者态度:+
It was once believed that the brain was independent of metabolic processes occurring elsewhere in the body. In recent studies, however, we have discovered that the production and release in brain neurons of the neurotransmitter serotonin (neurotransmitters are compounds that neurons use to transmit signals to other cells) depend directly on the food that the body processes.
Our first studies sought to determine whether the increase in serotonin observed in rats given a large injection of the amino acid tryptophan might also occur after rats ate meals that change tryptophan levels in the blood. We found that, immediately after the rats began to eat, parallel elevations occurred in blood tryptophan, brain tryptophan, and brain serotonin levels. These findings suggested that the production and release of serotonin in brain neurons were normally coupled with blood-tryptophan increases. In later studies we found that injecting insulin into a rat’s bloodstream also caused parallel elevations in blood and brain tryptophan levels and in serotonin levels. We then decided to see whether the secretion of the animal’s own insulin similarly affected serotonin production. We gave the rats a carbohydrate-containing meal that we knew would elicit insulin secretion. As we had hypothesized, the blood tryptophan level and the concentrations of tryptophan and of serotonin in the brain increased after the meal.
Surprisingly, however, when we added a large amount of protein to the meal, brain tryptophan and serotonin levels fell. Since protein contains tryptophan, why should it depress brain tryptophan levels? The answer lies in the mechanism that provides blood tryptophan to the brain cells. This same mechanism also provides the brain cells with other amino acids found in protein, such as tyrosine and leucine. The consumption of protein increases blood concentration of the other amino acids much more, proportionately, than it does that of tryptophan. The more protein is in a meal, the lower is the ratio of the resulting blood-tryptophan concentration to the concentration of competing amino acids, and the more slowly is tryptophan provided to the brain. Thus the more protein in a meal, the less serotonin subsequently produced and released.
Questions 82–90 refer to the passage above.
82. Which of the following titles best summarizes the contents of the passage? (主旨)
(A) Neurotransmitters: Their Crucial Function in Cellular Communication
(B) Diet and Survival: An Old Relationship Reexamined
(C) The Blood Supply and the Brain: A Reciprocal Dependence
(D) Amino Acids and Neurotransmitters: The Connection between Serotonin Levels and Tyrosine
(E) The Effects of Food Intake on the Production and Release of Serotonin: Some Recent Findings

83. According to the passage, the speed with which tryptophan is provided to the brain cells of a rat varies with the (细节)
(A) amount of protein present in a meal
(B) concentration of serotonin in the brain before a meal
(C) concentration of leucine in the blood rather than with the concentration of tyrosine in the blood after a meal
(D) concentration of tryptophan in the brain before a meal
(E) number of serotonin-containing neurons

84. According to the passage, when the authors began their first studies, they were aware that (细节)
(A) they would eventually need to design experiments that involved feeding rats high concentrations of protein
(B) tryptophan levels in the blood were difficult to monitor with accuracy
(C) serotonin levels increased after rats were fed meals rich in tryptophan
(D) there were many neurotransmitters whose production was dependent on metabolic processes elsewhere in the body
(E) serotonin levels increased after rats were injected with a large amount of tryptophan

85. According to the passage, one reason that the authors gave rats carbohydrates was to (细节)
(A) depress the rats’ tryptophan levels
(B) prevent the rats from contracting diseases
(C) cause the rats to produce insulin
(D) demonstrate that insulin is the most important substance secreted by the body
(E) compare the effect of carbohydrates with the effect of proteins

86. According to the passage, the more protein a rat consumes, the lower will be the (细节)
(A) ratio of the rat’s blood-tryptophan concentration to the amount of serotonin produced and released in the rat’s brain
(B) ratio of the rat’s blood-tryptophan concentration to the concentration in its blood of the other amino acids contained in the protein
(C) ratio of the rat’s blood-tyrosine concentration to its blood-leucine concentration
(D) number of neurotransmitters of any kind that the rat will produce and release
(E) number of amino acids the rat’s blood will contain

87. The authors’ discussion of the “mechanism that provides blood tryptophan to the brain cells” (lines 34–35) is meant to (结构)
(A) stimulate further research studies
(B) summarize an area of scientific investigation
(C) help explain why a particular research finding was obtained
(D) provide supporting evidence for a controversial scientific theory
(E) refute the conclusions of a previously mentioned research study

88. According to the passage, an injection of insulin was most similar in its effect on rats to an injection of (细节)
(A) tyrosine
(B) leucine
(C) blood
(D) tryptophan
(E) protein

89. It can be inferred from the passage that which of the following would be LEAST likely to be a potential source of aid to a patient who was not adequately producing and releasing serotonin? (推论)
(A) Meals consisting almost exclusively of protein
(B) Meals consisting almost exclusively of carbohydrates
(C) Meals that would elicit insulin secretion
(D) Meals that had very low concentrations of tyrosine
(E) Meals that had very low concentrations of leucine

90. It can be inferred from the passage that the authors initially held which of the following hypotheses about what would happen when they fed large amounts of protein to rats? (推论)
(A) The rats’ brain serotonin levels would not decrease.
(B) The rats’ brain tryptophan levels would decrease.
(C) The rats’ tyrosine levels would increase less quickly than would their leucine levels.
(D) The rats would produce more insulin.
(E) The rats would produce neurotransmitters other than serotonin.
作者: s    时间: 2010-11-22 21:25

Passage 17
题材类型:社会科学
结构类型:结论说明型
组织方式:总分,介绍著作=>分析著作=>结论1=>结论2
主要对象:women’s electoral participation
作者态度:±
In 1955 Maurice Duverger published The Political Role of Women, the first behavioralist, multinational comparison of women’s electoral participation ever to use election data and survey data together. His study analyzed women’s patterns of voting, political candidacy, and political activism in four European countries during the first half of the twentieth century. Duverger’s research findings were that women voted somewhat less frequently than men (the difference narrowing the longer women had the vote) and were slightly more conservative.
Duverger’s work set an early standard for the sensitive analysis of women’s electoral activities. Moreover, to Duverger’s credit, he placed his findings in the context of many of the historical processes that had shaped these activities. However, since these contexts have changed over time, Duverger’s approach has proved more durable than his actual findings. In addition, Duverger’s discussion of his findings was hampered by his failure to consider certain specific factors important to women’s electoral participation at the time he collected his data: the influence of political regimes, the effects of economic factors, and the ramifications of political and social relations between women and men. Given this failure, Duverger’s study foreshadowed the enduring limitations of the behavioralist approach to the multinational study of women’s political participation.
Questions 91–96 refer to the passage above.
91. The primary purpose of the passage is to (主旨)
(A) evaluate a research study
(B) summarize the history of a research area
(C) report new research findings
(D) reinterpret old research findings
(E) reconcile conflicting research findings

92. According to the passage, Duverger’s study was unique in 1955 in that it (细节)
(A) included both election data and survey data
(B) gathered data from sources never before used in political studies
(C) included an analysis of historical processes
(D) examined the influence on voting behavior of the relationships between women and men
(E) analyzed not only voting and political candidacy but also other political activities

93. Which of the following characteristics of a country is most clearly an example of a factor that Duverger, as described in the passage, failed to consider in his study? (细节)
(A) A large population
(B) A predominantly Protestant population
(C) A predominantly urban population
(D) A one-party government
(E) Location in the heart of Europe

94. The author implies that Duverger’s actual findings are (推论)
(A) limited because they focus on only four countries
(B) inaccurate in their description of the four countries in the early 1950s
(C) out-of-date in that they are inapplicable in the four countries today
(D) flawed because they are based on unsound data
(E) biased by Duverger’s political beliefs

95. The passage implies that, in comparing four European countries, Duverger found that the voting rates of women and men were most different in the country in which women (推论)
(A) were most politically active
(B) ran for office most often
(C) held the most conservative political views
(D) had the most egalitarian relations with men
(E) had possessed the right to vote for the shortest time

96. The author implies that some behavioralist research involving the multinational study of women’s political participation that followed Duverger’s study did which of the following? (推论)
(A) Ignored Duverger’s approach
(B) Suffered from faults similar to those in Duverger’s study
(C) Focused on political activism
(D) Focused on the influences of political regimes
(E) Focused on the political and social relations between women and men
作者: s    时间: 2010-11-22 21:26

Passage 18
题材类型:经济管理
结构类型:结论说明型
组织方式:总分,发现=>错误解释=>分析=>正确解释=>补充
主要对象:senior managers
作者态度:±
The majority of successful senior managers do not closely follow the classical rational model of first clarifying goals, assessing the problem, formulating options, estimating likelihoods of success, making a decision, and only then taking action to implement the decision. Rather, in their day-by-day tactical maneuvers, these senior executives rely on what is vaguely termed “intuition” to manage a network of interrelated problems that require them to deal with ambiguity, inconsistency, novelty, and surprise; and to integrate action into the process of thinking.
Generations of writers on management have recognized that some practicing managers rely heavily on intuition. In general, however, such writers display a poor grasp of what intuition is. Some see it as the opposite of rationality; others view it as an excuse for capriciousness.
Isenberg’s recent research on the cognitive processes of senior managers reveals that managers’ intuition is neither of these. Rather, senior managers use intuition in at least five distinct ways. First, they intuitively sense when a problem exists. Second, managers rely on intuition to perform well-learned behavior patterns rapidly. This intuition is not arbitrary or irrational, but is based on years of painstaking practice and hands-on experience that build skills. A third function of intuition is to synthesize isolated bits of data and practice into an integrated picture, often in an “Aha!” experience. Fourth, some managers use intuition as a check on the results of more rational analysis. Most senior executives are familiar with the formal decision analysis models and tools, and those who use such systematic methods for reaching decisions are occasionally leery of solutions suggested by these methods which run counter to their sense of the correct course of action. Finally, managers can use intuition to bypass in-depth analysis and move rapidly to engender a plausible solution. Used in this way, intuition is an almost instantaneous cognitive process in which a manager recognizes familiar patterns.
One of the implications of the intuitive style of executive management is that “thinking” is inseparable from acting. Since managers often “know” what is right before they can analyze and explain it, they frequently act first and explain later. Analysis is inextricably tied to action in thinking/acting cycles, in which managers develop thoughts about their companies and organizations not by analyzing a problematic situation and then acting, but by acting and analyzing in close concert. Given the great uncertainty of many of the management issues that they face, senior managers often instigate a course of action simply to learn more about an issue. They then use the results of the action to develop a more complete understanding of the issue. One implication of thinking/acting cycles is that action is often part of defining the problem, not just of implementing the solution.
Questions 97–102 refer to the passage above.
97. According to the passage, senior managers use intuition in all of the following ways EXCEPT to (细节)
(A) speed up the creation of a solution to a problem
(B) identify a problem
(C) bring together disparate facts
(D) stipulate clear goals
(E) evaluate possible solutions to a problem

98. The passage suggests which of the following about the “writers on management” mentioned in line 12? (推论)
(A) They have criticized managers for not following the classical rational model of decision analysis.
(B) They have not based their analyses on a sufficiently large sample of actual managers.
(C) They have relied in drawing their conclusions on what managers say rather than on what managers do.
(D) They have misunderstood how managers use intuition in making business decisions.
(E) They have not acknowledged the role of intuition in managerial practice.

99. Which of the following best exemplifies “an ‘Aha!’ experience” (line 30) as it is presented in the passage? (应用)
(A) A manager risks taking an action whose outcome is unpredictable to discover whether the action changes the problem at hand.
(B) A manager performs well-learned and familiar behavior patterns in creative and uncharacteristic ways to solve a problem.
(C) A manager suddenly connects seemingly unrelated facts and experiences to create a pattern relevant to the problem at hand.
(D) A manager rapidly identifies the methodology used to compile data yielded by systematic analysis.
(E) A manager swiftly decides which of several sets of tactics to implement in order to deal with the contingencies suggested by a problem.

100. According to the passage, the classical model of decision analysis includes all of the following EXCEPT (推论)
(A) evaluation of a problem
(B) creation of possible solutions to a problem
(C) establishment of clear goals to be reached by the decision
(D) action undertaken in order to discover more information about a problem
(E) comparison of the probable effects of different solutions to a problem

101. It can be inferred from the passage that which of the following would most probably be one major difference in behavior between Manager X, who uses intuition to reach decisions, and Manager Y, who uses only formal decision analysis? (应用)
(A) Manager X analyzes first and then acts; Manager Y does not.
(B) Manager X checks possible solutions to a problem by systematic analysis; Manager Y does not.
(C) Manager X takes action in order to arrive at the solution to a problem; Manager Y does not.
(D) Manager Y draws on years of hands-on experience in creating a solution to a problem; Manager X does not.
(E) Manager Y depends on day-to-day tactical maneuvering; Manager X does not.

102. The passage provides support for which of the following statements? (逻辑)
(A) Managers who rely on intuition are more successful than those who rely on formal decision analysis.
(B) Managers cannot justify their intuitive decisions.
(C) Managers’ intuition works contrary to their rational and analytical skills.
(D) Logical analysis of a problem increases the number of possible solutions.
(E) Intuition enables managers to employ their practical experience more efficiently.
作者: s    时间: 2010-11-22 21:26

Passage 19
题材类型:自然科学
结构类型:结论说明型
组织方式:平行,建议1=>理由1=>建议2=>理由2
主要对象:clinical trials
作者态度:+
Frazier and Mosteller assert that medical research could be improved by a move toward larger, simpler clinical trials of medical treatments. Currently, researchers collect far more background information on patients than is strictly required for their trials—substantially more than hospitals collect—thereby escalating costs of data collection, storage, and analysis. Although limiting information collection could increase the risk that researchers will overlook facts relevant to a study, Frazier and Mosteller contend that such risk, never entirely eliminable from research, would still be small in most studies. Only in research on entirely new treatments are new and unexpected variables likely to arise.
Frazier and Mosteller propose not only that researchers limit data collection on individual patients but also that researchers enroll more patients in clinical trials, thereby obtaining a more representative sample of the total population with the disease under study. Often researchers restrict study participation to patients who have no ailments besides those being studied. A treatment judged successful under these ideal conditions can then be evaluated under normal conditions. Broadening the range of trial participants, Frazier and Mosteller suggest, would enable researchers to evaluate a treatment’s efficacy for diverse patients under various conditions and to evaluate its effectiveness for different patient subgroups. For example, the value of a treatment for a progressive disease may vary according to a patient’s stage of disease. Patients’ ages may also affect a treatment’s efficacy.
Questions 103–107 refer to the passage above.
103. The passage is primarily concerned with (主旨)
(A) identifying two practices in medical research that may affect the accuracy of clinical trials
(B) describing aspects of medical research that tend to drive up costs
(C) evaluating an analysis of certain shortcomings of current medical research practices
(D) describing proposed changes to the ways in which clinical trials are conducted
(E) explaining how medical researchers have traditionally conducted clinical trials and how such trials are likely to change

104. Which of the following can be inferred from the passage about a study of the category of patients referred to in lines 21–23? (推论)
(A) Its findings might have limited applicability.
(B) It would be prohibitively expensive in its attempt to create ideal conditions.
(C) It would be the best way to sample the total population of potential patients.
(D) It would allow researchers to limit information collection without increasing the risk that important variables could be overlooked.
(E) Its findings would be more accurate if it concerned treatments for a progressive disease than if it concerned treatments for a nonprogressive disease.

105. It can be inferred from the passage that a study limited to patients like those mentioned in lines 21–23 would have which of the following advantages over the kind of study proposed by Frazier and Mosteller? (推论)
(A) It would yield more data and its findings would be more accurate.
(B) It would cost less in the long term, though it would be more expensive in its initial stages.
(C) It would limit the number of variables researchers would need to consider when evaluating the treatment under study.
(D) It would help researchers to identify subgroups of patients with secondary conditions that might also be treatable.
(E) It would enable researchers to assess the value of an experimental treatment for the average patient.

106. The author mentions patients’ ages (line 33) primarily in order to (评价)
(A) identify the most critical variable differentiating subgroups of patients
(B) cast doubt on the advisability of implementing Frazier and Mosteller’s proposals about medical research
(C) indicate why progressive diseases may require different treatments at different stages
(D) illustrate a point about the value of enrolling a wide range of patients in clinical trials
(E) substantiate an argument about the problems inherent in enrolling large numbers of patients in clinical trials

107. According to the passage, which of the following describes a result of the way in which researchers generally conduct clinical trials? (细节)
(A) They expend resources on the storage of information likely to be irrelevant to the study they are conducting.
(B) They sometimes compromise the accuracy of their findings by collecting and analyzing more information than is strictly required for their trials.
(C) They avoid the risk of overlooking variables that might affect their findings, even though doing so raises their research costs.
(D) Because they attempt to analyze too much information, they overlook facts that could emerge as relevant to their studies.
(E) In order to approximate the conditions typical of medical treatment, they base their methods of information collection on those used by hospitals.
作者: s    时间: 2010-11-22 21:27

Passage 20
题材类型:自然科学
结构类型:结论说明型
组织方式:总分,新理论=>与旧理论比较=>意义=>原理=>应用
主要对象:minerals
作者态度:+
According to a recent theory, Archean-age gold-quartz vein systems were formed more than two billion years ago from magmatic fluids that originated from molten granite-like bodies deep beneath the surface of the Earth. This theory is contrary to the widely held view that the systems were deposited from metamorphic fluids, that is, from fluids that formed during the dehydration of wet sedimentary rocks.
The recently developed theory has considerable practical importance. Most of the gold deposits discovered during the original gold rushes were exposed at the Earth’s surface and were found because they had shed trails of alluvial gold that were easily traced by simple prospecting methods. Although these same methods still lead to an occasional discovery, most deposits not yet discovered have gone undetected because they are buried and have no surface expression.
The challenge in exploration is therefore to unravel the subsurface geology of an area and pinpoint the position of buried minerals. Methods widely used today include analysis of aerial images that yield a broad geological overview; geophysical techniques that provide data on the magnetic, electrical, and mineralogical properties of the rocks being investigated; and sensitive chemical tests that are able to detect the subtle chemical halos that often envelop mineralization. However, none of these high-technology methods are of any value if the sites to which they are applied have never mineralized, and to maximize the chances of discovery the explorer must therefore pay particular attention to selecting the ground formations most likely to be mineralized. Such ground selection relies to varying degrees on conceptual models, which take into account theoretical studies of relevant factors.
These models are constructed primarily from empirical observations of known mineral deposits and from theories of ore-forming processes. The explorer uses the models to identify those geological features that are critical to the formation of the mineralization being modeled, and then tries to select areas for exploration that exhibit as many of the critical features as possible.
Questions 108–114 refer to the passage above.
108. The author is primarily concerned with (主旨)
(A) advocating a return to an older methodology
(B) explaining the importance of a recent theory
(C) enumerating differences between two widely used methods
(D) describing events leading to a discovery
(E) challenging the assumptions on which a theory is based

109. According to the passage, the widely held view of Archean-age gold-quartz vein systems is that such systems (细节)
(A) were formed from metamorphic fluids
(B) originated in molten granite-like bodies
(C) were formed from alluvial deposits
(D) generally have surface expression
(E) are not discoverable through chemical tests

110. The passage implies that which of the following steps would be the first performed by explorers who wish to maximize their chances of discovering gold? (推论)
(A) Surveying several sites known to have been formed more than two billion years ago
(B) Limiting exploration to sites known to have been formed from metamorphic fluid
(C) Using an appropriate conceptual model to select a site for further exploration
(D) Using geophysical methods to analyze rocks over a broad area
(E) Limiting exploration to sites where alluvial gold has previously been found

111. Which of the following statements about discoveries of gold deposits is supported by information in the passage? (细节)
(A) The number of gold discoveries made annually has increased between the time of the original gold rushes and the present.
(B) New discoveries of gold deposits are likely to be the result of exploration techniques designed to locate buried mineralization.
(C) It is unlikely that newly discovered gold deposits will ever yield as much as did those deposits discovered during the original gold rushes.
(D) Modern explorers are divided on the question of the utility of simple prospecting methods as a source of new discoveries of gold deposits.
(E) Models based on the theory that gold originated from magmatic fluids have already led to new discoveries of gold deposits.

112. It can be inferred from the passage that which of the following is easiest to detect? (应用)
(A) A gold-quartz vein system originating in magmatic fluids
(B) A gold-quartz vein system originating in metamorphic fluids
(C) A gold deposit that is mixed with granite
(D) A gold deposit that has shed alluvial gold
(E) A gold deposit that exhibits chemical halos

113. The theory mentioned in lines 1–5 relates to the conceptual models discussed in the passage in which of the following ways? (结构)
(A) It may furnish a valid account of ore-forming processes, and, hence, can support conceptual models that have great practical significance.
(B) It suggests that certain geological formations, long believed to be mineralized, are in fact mineralized, thus confirming current conceptual models.
(C) It suggests that there may not be enough similarity across Archean-age gold-quartz vein systems to warrant the formulation of conceptual models.
(D) It corrects existing theories about the chemical halos of gold deposits, and thus provides a basis for correcting current conceptual models.
(E) It suggests that simple prospecting methods still have a higher success rate in the discovery of gold deposits than do more modern methods.

114. According to the passage, methods of exploring for gold that are widely used today are based on which of the following facts? (细节)
(A) Most of the Earth’s remaining gold deposits are still molten.
(B) Most of the Earth’s remaining gold deposits are exposed at the surface.
(C) Most of the Earth’s remaining gold deposits are buried and have no surface expression.
(D) Only one type of gold deposit warrants exploration, since the other types of gold deposits are found in regions difficult to reach.
(E) Only one type of gold deposit warrants exploration, since the other types of gold deposits are unlikely to yield concentrated quantities of gold.
作者: s    时间: 2010-11-22 21:27

Passage 21
题材类型:自然科学
结构类型:问题回答型
组织方式:总分,提出问题=>新发现=>结论
主要对象:universe expansion
作者态度:+
After evidence was obtained in the 1920s that the universe is expanding, it became reasonable to ask: Will the universe continue to expand indefinitely, or is there enough mass in it for the mutual attraction of its constituents to bring this expansion to a halt? It can be calculated that the critical density of matter needed to brake the expansion and “close” the universe is equivalent to three hydrogen atoms per cubic meter. But the density of the observable universe—luminous matter in the form of galaxies—comes to only a fraction of this. If the expansion of the universe is to stop, there must be enough invisible matter in the universe to exceed the luminous matter in density by a factor of roughly 70.
Our contribution to the search for this “missing matter” has been to study the rotational velocity of galaxies at various distances from their center of rotation. It has been known for some time that outside the bright nucleus of a typical spiral galaxy luminosity falls off rapidly with distance from the center. If luminosity were a true indicator of mass, most of the mass would be concentrated toward the center. Outside the nucleus the rotational velocity would decrease geometrically with distance from the center, in conformity with Kepler’s law. Instead we have found that the rotational velocity in spiral galaxies either remains constant with increasing distance from the center or increases slightly. This unexpected result indicates that the falloff in luminous mass with distance from the center is balanced by an increase in nonluminous mass.
Our findings suggest that as much as 90 percent of the mass of the universe is not radiating at any wavelength with enough intensity to be detected on the Earth. Such dark matter could be in the form of extremely dim stars of low mass, of large planets like Jupiter, or of black holes, either small or massive. While it has not yet been determined whether this mass is sufficient to close the universe, some physicists consider it significant that estimates are converging on the critical value.
Questions 115–119 refer to the passage above.
115. The passage is primarily concerned with (主旨)
(A) defending a controversial approach
(B) criticizing an accepted view
(C) summarizing research findings
(D) contrasting competing theories
(E) describing an innovative technique

116. The authors’ study indicates that, in comparison with the outermost regions of a typical spiral galaxy, the region just outside the nucleus can be characterized as having (推论)
(A) higher rotational velocity and higher luminosity
(B) lower rotational velocity and higher luminosity
(C) lower rotational velocity and lower luminosity
(D) similar rotational velocity and higher luminosity
(E) similar rotational velocity and similar luminosity

117. The authors’ suggestion that “as much as 90 percent of the mass of the universe is not radiating at any wavelength with enough intensity to be detected on the Earth” (lines 34–37) would be most weakened if which of the following were discovered to be true? (逻辑)
(A) Spiral galaxies are less common than types of galaxies that contain little nonluminous matter.
(B) Luminous and nonluminous matter are composed of the same basic elements.
(C) The bright nucleus of a typical spiral galaxy also contains some nonluminous matter.
(D) The density of the observable universe is greater than most previous estimates have suggested.
(E) Some galaxies do not rotate or rotate too slowly for their rotational velocity to be measured.

118. It can be inferred from information presented in the passage that if the density of the universe were equivalent to significantly less than three hydrogen atoms per cubic meter, which of the following would be true as a consequence? (推论)
(A) Luminosity would be a true indicator of mass.
(B) Different regions in spiral galaxies would rotate at the same velocity.
(C) The universe would continue to expand indefinitely.
(D) The density of the invisible matter in the universe would have to be more than 70 times the density of the luminous matter.
(E) More of the invisible matter in spiral galaxies would have to be located in their nuclei than in their outer regions.

119. The authors propose all of the following as possibly contributing to the “missing matter” in spiral galaxies EXCEPT (细节)
(A) massive black holes
(B) small black holes
(C) small, dim stars
(D) massive stars
(E) large planets
作者: s    时间: 2010-11-22 21:28

Passage 22
题材类型:社会科学
结构类型:结论说明型
组织方式:总分,甲方观点=>乙方观点=>乙方缺陷=>甲方分析
主要对象:technological determinism and social constructivism
作者态度:±
Jon Clark’s study of the effect of the modernization of a telephone exchange on exchange maintenance work and workers is a solid contribution to a debate that encompasses two lively issues in the history and sociology of technology: technological determinism and social constructivism.
Clark makes the point that the characteristics of a technology have a decisive influence on job skills and work organization. Put more strongly, technology can be a primary determinant of social and managerial organization. Clark believes this possibility has been obscured by the recent sociological fashion, exemplified by Braverman’s analysis, that emphasizes the way machinery reflects social choices. For Braverman, the shape of a technological system is subordinate to the manager’s desire to wrest control of the labor process from the workers. Technological change is construed as the outcome of negotiations among interested parties who seek to incorporate their own interests into the design and configuration of the machinery. This position represents the new mainstream called social constructivism.
The constructivists gain acceptance by misrepresenting technological determinism: technological determinists are supposed to believe, for example, that machinery imposes appropriate forms of order on society. The alternative to constructivism, in other words, is to view technology as existing outside society, capable of directly influencing skills and work organization.
Clark refutes the extremes of the constructivists by both theoretical and empirical arguments. Theoretically he defines “technology” in terms of relationships between social and technical variables. Attempts to reduce the meaning of technology to cold, hard metal are bound to fail, for machinery is just scrap unless it is organized functionally and supported by appropriate systems of operation and maintenance. At the empirical level Clark shows how a change at the telephone exchange from maintenance-intensive electromechanical switches to semielectronic switching systems altered work tasks, skills, training opportunities, administration, and organization of workers. Some changes Clark attributes to the particular way management and labor unions negotiated the introduction of the technology, whereas others are seen as arising from the capabilities and nature of the technology itself. Thus Clark helps answer the question: “When is social choice decisive and when are the concrete characteristics of technology more important?”
Questions 120–127 refer to the passage above.
120. The primary purpose of the passage is to (主旨)
(A) advocate a more positive attitude toward technological change
(B) discuss the implications for employees of the modernization of a telephone exchange
(C) consider a successful challenge to the constructivist view of technological change
(D) challenge the position of advocates of technological determinism
(E) suggest that the social causes of technological change should be studied in real situations

121. Which of the following statements about the modernization of the telephone exchange is supported by information in the passage? (细节)
(A) The new technology reduced the role of managers in labor negotiations.
(B) The modernization was implemented without the consent of the employees directly affected by it.
(C) The modernization had an impact that went significantly beyond maintenance routines.
(D) Some of the maintenance workers felt victimized by the new technology.
(E) The modernization gave credence to the view of advocates of social constructivism.

122. Which of the following most accurately describes Clark’s opinion of Braverman’s position? (推论)
(A) He respects its wide-ranging popularity.
(B) He disapproves of its misplaced emphasis on the influence of managers.
(C) He admires the consideration it gives to the attitudes of the workers affected.
(D) He is concerned about its potential to impede the implementation of new technologies.
(E) He is sympathetic to its concern about the impact of modern technology on workers.

123. The information in the passage suggests that which of the following statements from hypothetical sociological studies of change in industry most clearly exemplifies the social constructivists’ version of technological determinism? (应用)
(A) It is the available technology that determines workers’ skills, rather than workers’ skills influencing the application of technology.
(B) All progress in industrial technology grows out of a continuing negotiation between technological possibility and human need.
(C) Some organizational change is caused by people; some is caused by computer chips.
(D) Most major technological advances in industry have been generated through research and development.
(E) Some industrial technology eliminates jobs, but educated workers can create whole new skills areas by the adaptation of the technology.

124. The information in the passage suggests that Clark believes that which of the following would be true if social constructivism had not gained widespread acceptance? (推论)
(A) Businesses would be more likely to modernize without considering the social consequences of their actions.
(B) There would be greater understanding of the role played by technology in producing social change.
(C) Businesses would be less likely to understand the attitudes of employees affected by modernization.
(D) Modernization would have occurred at a slower rate.
(E) Technology would have played a greater part in determining the role of business in society.

125. According to the passage, constructivists employed which of the following to promote their argument? (细节)
(A) Empirical studies of business situations involving technological change
(B) Citation of managers supportive of their position
(C) Construction of hypothetical situations that support their view
(D) Contrasts of their view with a misstatement of an opposing view
(E) Descriptions of the breadth of impact of technological change

126. The author of the passage uses the expression “are supposed to” in line 27 primarily in order to (评价)
(A) suggest that a contention made by constructivists regarding determinists is inaccurate
(B) define the generally accepted position of determinists regarding the implementation of technology
(C) engage in speculation about the motivation of determinists
(D) lend support to a comment critical of the position of determinists
(E) contrast the historical position of determinists with their position regarding the exchange modernization

127. Which of the following statements about Clark’s study of the telephone exchange can be inferred from information in the passage? (推论)
(A) Clark’s reason for undertaking the study was to undermine Braverman’s analysis of the function of technology.
(B) Clark’s study suggests that the implementation of technology should be discussed in the context of conflict between labor and management.
(C) Clark examined the impact of changes in the technology of switching at the exchange in terms of overall operations and organization.
(D) Clark concluded that the implementation of new switching technology was equally beneficial to management and labor.
(E) Clark’s analysis of the change in switching systems applies only narrowly to the situation at the particular exchange that he studied.
作者: s    时间: 2010-11-22 21:28

Passage 23
题材类型:自然科学•生命科学
结构类型:问题回答型
组织方式:总分,提出问题=>回答问题=>解释1=>解释2
主要对象:hormones
作者态度:+
All the cells in a particular plant start out with the same complement of genes. How then can these cells differentiate and form structures as different as roots, stems, leaves, and fruits? The answer is that only a small subset of the genes in a particular kind of cell are expressed, or turned on, at a given time. This is accomplished by a complex system of chemical messengers that in plants include hormones and other regulatory molecules. Five major hormones have been identified: auxin, abscisic acid, cytokinin, ethylene, and gibberellin. Studies of plants have now identified a new class of regulatory molecules called oligosaccharins.
Unlike the oligosaccharins, the five well-known plant hormones are pleiotropic rather than specific; that is, each has more than one effect on the growth and development of plants. The five have so many simultaneous effects that they are not very useful in artificially controlling the growth of crops. Auxin, for instance, stimulates the rate of cell elongation, causes shoots to grow up and roots to grow down, and inhibits the growth of lateral shoots. Auxin also causes the plant to develop a vascular system, to form lateral roots, and to produce ethylene.
The pleiotropy of the five well-studied plant hormones is somewhat analogous to that of certain hormones in animals. For example, hormones from the hypothalamus in the brain stimulate the anterior lobe of the pituitary gland to synthesize and release many different hormones, one of which stimulates the release of hormones from the adrenal cortex. These hormones have specific effects on target organs all over the body. One hormone stimulates the thyroid gland, for example, another the ovarian follicle cells, and so forth. In other words, there is a hierarchy of hormones.
Such a hierarchy may also exist in plants. Oligosaccharins are fragments of the cell wall released by enzymes: different enzymes release different oligosaccharins. There are indications that pleiotropic plant hormones may actually function by activating the enzymes that release these other, more specific chemical messengers from the cell wall.
Questions 128–133 refer to the passage above.
128. According to the passage, the five well-known plant hormones are not useful in controlling the growth of crops because (细节)
(A) it is not known exactly what functions the hormones perform
(B) each hormone has various effects on plants
(C) none of the hormones can function without the others
(D) each hormone has different effects on different kinds of plants
(E) each hormone works on only a small subset of a cell’s genes at any particular time

129. The passage suggests that the place of hypothalamic hormones in the hormonal hierarchies of animals is similar to the place of which of the following in plants? (推论)
(A) Plant cell walls
(B) The complement of genes in each plant cell
(C) A subset of a plant cell’s gene complement
(D) The five major hormones
(E) The oligosaccharins

130. The passage suggests that which of the following is a function likely to be performed by an oligosaccharin? (推论)
(A) To stimulate a particular plant cell to become part of a plant’s root system
(B) To stimulate the walls of a particular cell to produce other oligosaccharins
(C) To activate enzymes that release specific chemical messengers from plant cell walls
(D) To duplicate the gene complement in a particular plant cell
(E) To produce multiple effects on a particular subsystem of plant cells

131. The author mentions specific effects that auxin has on plant development in order to illustrate the (结构)
(A) point that some of the effects of plant hormones can be harmful
(B) way in which hormones are produced by plants
(C) hierarchical nature of the functioning of plant hormones
(D) differences among the best-known plant hormones
(E) concept of pleiotropy as it is exhibited by plant hormones

132. According to the passage, which of the following best describes a function performed by oligosaccharins? (细节)
(A) Regulating the daily functioning of a plant’s cells
(B) Interacting with one another to produce different chemicals
(C) Releasing specific chemical messengers from a plant’s cell walls
(D) Producing the hormones that cause plant cells to differentiate to perform different functions
(E) Influencing the development of a plant’s cells by controlling the expression of the cells’ genes

133. The passage suggests that, unlike the pleiotropic hormones, oligosaccharins could be used effectively to (推论)
(A) trace the passage of chemicals through the walls of cells
(B) pinpoint functions of other plant hormones
(C) artificially control specific aspects of the development of crops
(D) alter the complement of genes in the cells of plants
(E) alter the effects of the five major hormones on plant development
作者: s    时间: 2010-11-22 21:29

Passage 24
题材类型:社会科学•美国黑人
结构类型:新老观点型
组织方式:总分,旧观点=>旧观点解释=>新观点=>新观点解释
主要对象:Great Migration
作者态度:±
In the two decades between 1910 and 1930, more than ten percent of the black population of the United States left the South, where the preponderance of the black population had been located, and migrated to northern states, with the largest number moving, it is claimed, between 1916 and 1918. It has been frequently assumed, but not proved, that the majority of the migrants in what has come to be called the Great Migration came from rural areas and were motivated by two concurrent factors: the collapse of the cotton industry following the boll weevil infestation, which began in 1898, and increased demand in the North for labor following the cessation of European immigration caused by the outbreak of the First World War in 1914. This assumption has led to the conclusion that the migrants’ subsequent lack of economic mobility in the North is tied to rural background, a background that implies unfamiliarity with urban living and a lack of industrial skills.
But the question of who actually left the South has never been rigorously investigated. Although numerous investigations document an exodus from rural southern areas to southern cities prior to the Great Migration, no one has considered whether the same migrants then moved on to northern cities. In 1910 more than 600,000 black workers, or ten percent of the black workforce, reported themselves to be engaged in “manufacturing and mechanical pursuits,” the federal census category roughly encompassing the entire industrial sector. The Great Migration could easily have been made up entirely of this group and their families. It is perhaps surprising to argue that an employed population could be enticed to move, but an explanation lies in the labor conditions then prevalent in the South.
About thirty-five percent of the urban black population in the South was engaged in skilled trades. Some were from the old artisan class of slavery—blacksmiths, masons, carpenters—which had had a monopoly of certain trades, but they were gradually being pushed out by competition, mechanization, and obsolescence. The remaining sixty-five percent, more recently urbanized, worked in newly developed industries—tobacco, lumber, coal and iron manufacture, and railroads. Wages in the South, however, were low, and black workers were aware, through labor recruiters and the black press, that they could earn more even as unskilled workers in the North than they could as artisans in the South. After the boll weevil infestation, urban black workers faced competition from the continuing influx of both black and white rural workers, who were driven to undercut the wages formerly paid for industrial jobs. Thus, a move north would be seen as advantageous to a group that was already urbanized and steadily employed, and the easy conclusion tying their subsequent economic problems in the North to their rural background comes into question.
Questions 134–139 refer to the passage above.
134. The author indicates explicitly that which of the following records has been a source of information in her investigation? (细节)
(A) United States Immigration Service reports from 1914 to 1930
(B) Payrolls of southern manufacturing firms between 1910 and 1930
(C) The volume of cotton exports between 1898 and 1910
(D) The federal census of 1910
(E) Advertisements of labor recruiters appearing in southern newspapers after 1910

135. In the passage, the author anticipates which of the following as a possible objection to her argument? (逻辑)
(A) It is uncertain how many people actually migrated during the Great Migration.
(B) The eventual economic status of the Great Migration migrants has not been adequately traced.
(C) It is not likely that people with steady jobs would have reason to move to another area of the country.
(D) It is not true that the term “manufacturing and mechanical pursuits” actually encompasses the entire industrial sector.
(E) Of the African American workers living in southern cities, only those in a small number of trades were threatened by obsolescence.

136. According to the passage, which of the following is true of wages in southern cities in 1910? (细节)
(A) They were being pushed lower as a result of increased competition.
(B) They had begun to rise so that southern industry could attract rural workers.
(C) They had increased for skilled workers but decreased for unskilled workers.
(D) They had increased in large southern cities but decreased in small southern cities.
(E) They had increased in newly developed industries but decreased in the older trades.

137. The author cites each of the following as possible influences in an African American worker’s decision to migrate north in the Great Migration EXCEPT (细节)
(A) wage levels in northern cities
(B) labor recruiters
(C) competition from rural workers
(D) voting rights in northern states
(E) the African American press

138. It can be inferred from the passage that the “easy conclusion” mentioned in line 58 is based on which of the following assumptions? (逻辑)
(A) People who migrate from rural areas to large cities usually do so for economic reasons.
(B) Most people who leave rural areas to take jobs in cities return to rural areas as soon as it is financially possible for them to do so.
(C) People with rural backgrounds are less likely to succeed economically in cities than are those with urban backgrounds.
(D) Most people who were once skilled workers are not willing to work as unskilled workers.
(E) People who migrate from their birthplaces to other regions of a country seldom undertake a second migration.

139. The primary purpose of the passage is to (主旨)
(A) support an alternative to an accepted methodology
(B) present evidence that resolves a contradiction
(C) introduce a recently discovered source of information
(D) challenge a widely accepted explanation
(E) argue that a discarded theory deserves new attention
作者: s    时间: 2010-11-22 21:29

二、阅读理解答案
1.C        2. D        3. B        4. E        5. B        6. D        7. D        8. A        9. B        10. E
11. E        12. D        13. E        14. D        15. D        16. C        17. A        18. B        19. B        20. B
21. A        22. D        23. E        24. D        25. A        26. E        27. C        28. C        29. C        30. E
31. C        32. B        33. A        34. C        35. B        36. C        37. B        38. D        39. B        40. E
41. C        42. C        43. D        44. D        45. A        46. E        47. B        48. A        49. A        50. C
51. B        52. E        53. E        54. A        55. A        56. D        57. D        58. C        59. B        60. A
61. A        62. B        63. E        64. B        65. D        66. E        67. D        68. D        69. B        70. D
71. D        72. B        73. C        74. B        75. E        76. D        77. E        78. B        79. D        80. C
81. C        82. E        83. A        84. E        85. C        86. B        87. C        88. D        89. A        90. A
91. A        92. A        93. D        94. C        95. E        96. B        97. D        98. D        99. C        100. D
101. C        102. E        103. D        104. A        105. C        106. D        107. A        108. B        109. A        110. C
111. B        112. D        113. A        114. C        115. C        116. D        117. A        118. C        119. D        120. C
121. C        122. B        123. A        124. B        125. D        126. A        127. C        128. B        129. D        130. A
131. E        132. E        133. C        134. D        135. C        136. A        137. D        138. C        139. D
作者: s    时间: 2010-11-22 21:30

GWD阅读理解
一、阅读理解试题
GWD-TN-14 Passage 1
题材类型:社会科学•美国妇女
结构类型:新老观点型
组织方式:总分,旧观点=>旧观点解释=>新观点=>新观点解释
主要对象:time spent on housework
作者态度:大负小正
The idea that equipping homes with electrical appliances and other “modern” household technologies would eliminate drudgery, save labor time, and increase leisure for women who were full-time home workers remained largely unchallenged until the women’s movement of the 1970’s spawned the groundbreaking and influential works of sociologist Joann Vanek and historian Ruth Cowan. Vanek analyzed 40 years of time- use surveys conducted by home economists to argue that electrical appliances and other modern household technologies reduced the effort required to perform specific tasks, but ownership of these appliances did not correlate with less time spent on housework by full-time home workers. In fact, time spent by these workers remained remarkably constant― at about 52 to 54 hours per week― from the 1920’s to the 1960’s, a period of significant change in household technology.  In surveying two centuries of household technology in the United States, Cowan argued that the “industrialization” of the home often resulted in more work for full-time home workers because the use of such devices as coal stoves, water pumps, and vacuum cleaners tended to reduce the workload of married-women’s helpers (husbands, sons, daughters, and servants) while promoting a more rigorous standard of housework.  The full-time home worker’s duties also shifted to include more household management, child care, and the post-Second World War phenomenon of being “Mom’s taxi.”

4. According to the passage, which of the following is true about the idea mentioned in line1? (信息题)
A.        It has been undermined by data found in time-use surveys conducted by home economists.
B.        It was based on a definition of housework that was explicitly rejected by Vanek and Cowan.
C.        It is more valid for the time period studied by Cowan than for the time period studied by Vanek.
D.        It is based on an underestimation of the time that married women spent on housework prior to the industrialization of the household.
E.        It inaccurately suggested that new household technologies would reduce the effort required to perform housework.

5. The passage is primarily concerned with(写法性主题题)
A.        analyzing a debate between two scholars
B.        challenging the evidence on which a new theory is based
C.        describing how certain scholars’ work countered a prevailing view
D.        presenting the research used to support a traditional theory
E.        evaluating the methodology used to study a particular issue

6. Which of the following best describes the function of the sentence in lines 21-26 (“In fact, time … in household technology”)? (举例题)
A.        It offers an alternative interpretation of a phenomenon described in the previous sentence (lines 12-20).
B.        It provides the specific evidence on which an argument described in the previous sentence (lines 12-20) is based.
C.        It shifts the focus of the argument developed earlier in the passage.
D.        It introduces evidence that has not been taken into account by Vanek and Cowan.
E.        It introduces a topic for discussion that will be developed in the rest of the passage.

7. The passage suggests that Vanek and Cowan would agree that modernizing household technology did not(信息题)
A.        reduce the workload of servants and other household helpers
B.        raise the standard of housework that women who were full-time home workers set for themselves
C.        decrease the effort required to perform household tasks
D.        reduce the time spent on housework by women who were full-time home workers
E.        result in a savings of money used for household maintenance
作者: s    时间: 2010-11-22 21:31

GWD-TN-14 Passage 2
题材类型:自然科学•生命科学
结构类型:结论说明型
组织方式:总分,定义=>结论=>实验=>解释
主要对象:episodic memory
代表人物:Clayton
作者态度:大正小负
The term “episodic memory” was introduced by Tulving to refer to what he considered a uniquely human capacity—the ability to recollect specific past events, to travel back into the past in one’s own mind—as distinct from the capacity simply to use information acquired through past experiences.  Subsequently, Clayton et al. developed criteria to test for episodic memory in animals.  According to these criteria, episodic memories are not of individual bits of information; they involve multiple components of a single event “bound” together.  Clayton sought to examine evidence of scrub jays’ accurate memory of “what,” “where,” and “when” information and their binding of this information.  In the wild, these birds store food for retrieval later during periods of food scarcity.  Clayton’s experiment required jays to remember the type, location, and freshness of stored food based on a unique learning event.  Crickets were stored in one location and peanuts in another.  Jays prefer crickets, but crickets degrade more quickly.  Clayton’s birds switched their preference from crickets to peanuts once the food had been stored for a certain length of time, showing that they retain information about the what, the where, and the when.  Such experiments cannot, however, reveal whether the birds were reexperiencing the past when retrieving the information.  Clayton acknowledged this by using the term “episodic-like” memory.

8. According to the passage, part of the evidence that scrub jays can bind information is that they(信息题)
A.        showed by their behavior that they were reexperiencing the past
B.        used information acquired through past experiences
C.        assessed the freshness of food that had been stored by other jays
D.        remembered what kind of food was stored in a particular location
E.        recollected single bits of information about sources of food

9. It can be inferred that the author of the passage and Clayton would both agree that(信息题)
A.        the food preferences of the scrub jays in Clayton’s experiment are difficult to explain
B.        the presence of episodic memory cannot be inferred solely on the basis of observable behavior
C.        Clayton’s experiment demonstrated that scrub jays do not reexperience the past but do exhibit episodic-like memory
D.        Tulving substantially underestimated the ability of animals to bind different kinds of information
E.        Clayton’s experiment had certain fundamental design flaws that make it difficult to draw any conclusions about scrub jay’s memories

10. In order for Clayton’s experiment to show that scrub jays have episodic-like memory, which of the following must be true in the experiment? (改善题)
A.        Some of the jays retrieved stored peanuts on the first occasion they were allowed to retrieve food.
B.        All the crickets were retrieved before any of the peanuts were.
C.        The peanuts were stored further away than the crickets.
D.        When a jay attempted to retrieve a cricket or a peanut, the jay was prevented from eating it.
E.        Throughout the experiment the jays were fed at levels typical of a time of scarcity.
作者: s    时间: 2010-11-22 21:31

GWD-TN-14 Passage 3
题材类型:社会科学•美国历史
结构类型:现象解释型
组织方式:总分,史实=>展开=>发展
主要对象:health care
代表人物:Gompers
作者态度:大负小正
In 1938, at the government-convened National Health Conference, organized labor emerged as a major proponent of legislation to guarantee universal health care in the United States.  The American Medical Association, representing physicians’ interests, argued for preserving physicians’ free-market prerogatives.  Labor activists countered these arguments by insisting that health care was a fundamental right that should be guaranteed by government programs.
The labor activists’ position represented a departure from the voluntarist view held until 1935 by leaders of the American Federation of labor (AFL), a leading affiliation of labor unions; the voluntarist view stressed workers’ right to freedom from government intrusions into their lives and represented national health insurance as a threat to workers’ privacy.  AFL president Samuel Gompers, presuming to speak for all workers, had positioned the AFL as a leading opponent of the proposals for national health insurance that were advocated beginning in 1915 by the American Association for Labor Legislation (AALL), an organization dedicated to the study and reform of labor laws.  Gompers’ opposition to national health insurance was partly principled, arising from the premise that governments under capitalism invariably served employers’, not workers’, interests. Gompers feared the probing of government bureaucrats into workers’ lives, as well as the possibility that government-mandated health insurance, financed in part by employers, could permit companies to require employee medical examinations that might be used to discharge disabled workers.
    Yet the AFL’s voluntarism had accommodated certain exceptions:  the AFL had supported government intervention on behalf of injured workers and child laborers.  AFL officials drew the line at national health insurance, however, partly out of concern for their own power.  The fact that AFL outsiders such as the AALL had taken the most prominent advocacy roles antagonized Gompers.  That this reform threatened union-sponsored benefit programs championed by Gompers made national health insurance even more objectionable.
    Indeed, the AFL leadership did face serious organizational divisions.  Many unionists, recognizing that union-run health programs covered only a small fraction of union members and that unions represented only a fraction of the nation’s workforce, worked to enact compulsory health insurance in their state legislatures.  This activism and the views underlying it came to prevail in the United States labor movement and in 1935 the AFL unequivocally reversed its position on health legislation.

24. The passage suggests which of the following about the voluntarist view held by leaders of the AFL regarding health care? (事实题)
A.        It was opposed by the AALL.
B.        It was shared by most unionists until 1935.
C.        It antagonized the American Medical Association.
D.        It maintained that employer-sponsored health care was preferable to union-run health programs.
E.        It was based on the premise that the government should protect child laborers but not adult workers.

25. The primary purpose of the passage is to(内容性主题题)
A.        account for a labor organization’s success in achieving a particular goal
B.        discuss how a labor organization came to reverse its position on a particular issue
C.        explain how disagreement over a particular issue eroded the power of a labor organization
D.        outline the arguments used by a labor organization’s leadership in a particular debate
E.        question the extent to which a labor organization changed its position on a particular issue

26. Which of the following best describes the function of the sentence in lines 42-45 (“Yet … child laborers”)? (举例题)
A.        It elaborates a point about why the AFL advocated a voluntarist approach to health insurance.
B.        It identifies issues on which the AFL took a view opposed to that of the AALL.
C.        It introduces evidence that appears to be inconsistent with the voluntarist view held by AFL leaders.
D.        It suggests that a view described in the previous sentence is based on faulty evidence.
E.        It indicates why a contradiction described in the previous paragraph has been overlooked by historians.

27. According to the passage, Gompers’ objection to national health insurance was based in part on his belief that(事实题)
A.        union-sponsored health programs were less expensive than government-sponsored programs
B.        most unionists were covered by and satisfied with union-sponsored health programs
C.        it would lead some employers to reduce company-sponsored benefits
D.        it could result in certain workers unfairly losing their jobs
E.        the AFL should distance itself from the views of the American Medical Association
作者: s    时间: 2010-11-22 21:32

GWD-TN-14 Passage 4
题材类型:自然科学•天文
结构类型:现象解释型
组织方式:总分,现象=>理论=>解释
主要对象:gravity anomaly
代表人物:
作者态度:正
(This passage was excerpted from material published in 1996.)
When a large body strikes a planet or moon, material is ejected, thereby creating a hole in the planet and a local deficit of mass. This deficit shows up as a gravity anomaly: the removal of the material that has been ejected to make the hole results in an area of slightly lower gravity than surrounding areas. One would therefore expect that all of the large multi-ring impact basins on the surface of Earth’s Moon would show such negative gravity anomalies, since they are, essentially, large holes in the lunar surface. Yet data collected in 1994 by the Clementine spacecraft show that many of these lunar basins have no anomalously low gravity and some even have anomalously high gravity. Scientists speculate that early in lunar history, when large impactors struck the Moon’s surface, causing millions of cubic kilometers of crustal debris to be ejected, denser material from the Moon’s mantle rose up beneath the impactors almost immediately, compensating for the ejected material and thus leaving no low gravity anomaly in the resulting basin. Later, however, as the Moon grew cooler and less elastic, rebound from large impactors would have been only partial and incomplete. Thus today such gravitational compensation probably would not occur: the outer layer of the Moon is too cold and stiff.

35. According to the passage, the gravitational compensation referred to in line 33 is caused by which of the following? (信息题)
A.        A deficit of mass resulting from the creation of a hole in the lunar surface
B.        The presence of material from the impactor in the debris created by its impact
C.        The gradual cooling and stiffening of the Moon’s outer layer
D.        The ejection of massive amounts of debris from the Moon’s crust
E.        The rapid upwelling of material from the lunar mantle

36. The passage suggests that if the scientists mentioned in line 19 are correct in their speculations, the large multi-ring impact basins on the Moon with the most significant negative gravity anomalies probably(信息题)
A.        were not formed early in the Moon’s history
B.        were not formed by the massive ejection of crustal debris
C.        are closely surrounded by other impact basins with anomalously low gravity
D.        were created by the impact of multiple large impactors
E.        were formed when the Moon was relatively elastic

37. Which of the following best describes the organization of the passage? (写法性主题题)
A.        An anomalous finding is cited, the data used to support that finding are analyzed, and the finding is modified.
B.        A theory about a phenomenon is introduced, data seeming to disprove that theory are analyzed, and the theory is rejected.
C.        A phenomenon is described, a finding relating to the phenomenon is discussed, and a possible explanation for that finding is offered.
D.        A debate among scientists regarding the explanation for a particular phenomenon is outlined, and one position in that debate is shown to be more persuasive.
E.        The observation of an astronomical event is described, and two schools of thought about the explanation for that event are discussed.
作者: s    时间: 2010-11-22 21:32

GWD-TN-15 Passage 1
题材类型:经济管理
结构类型:新老观点型
组织方式:总分,旧观点=>分析=>新观点
主要对象:labor theory of value
代表人物:Locke
作者态度:大负小正
Seventeenth-century philosopher John Locke stated that as much as 99 percent of the value of any useful product can be attributed to “the effects of labor.” For Locke’s intellectual heirs it was only a short step to the “labor theory of value,” whose formulators held that 100 percent of the value of any product is generated by labor (the human work needed to produce goods) and that therefore the employer who appropriates any part of the product’s value as profit is practicing theft.
Although human effort is required to produce goods for the consumer market, effort is also invested in making capital goods (tools, machines, etc.), which are used to facilitate the production of consumer goods.  In modern economies about one-third of the total output of consumer goods is attributable to the use of capital goods.  Approximately two-thirds of the income derived from this total output is paid out to workers as wages and salaries, the remaining third serving as compensation to the owners of the capital goods. Moreover, part of this remaining third is received by workers who are shareholders, pension beneficiaries, and the like. The labor theory of value systematically disregards the productive contribution of capital goods—a failing for which Locke must bear part of the blame.

7. According to the author of the passage, which of the following is true of the distribution of the income derived from the total output of consumer goods in a modern economy? (信息题)
A.        Workers receive a share of this income that is significantly smaller than the value of their labor as a contribution to total output.
B.        Owners of capital goods receive a share of this income that is significantly greater than the contribution to total output attributable to the use of capital goods.
C.        Owners of capital goods receive a share of this income that is no greater than the proportion of total output attributable to the use of capital goods.
D.        Owners of capital goods are not fully compensated for their investment because they pay out most of their share of this income to workers as wages and benefits.
E.        Workers receive a share of this income that is greater than the value of their labor because the labor theory of value overestimates their contribution to total output.

8. The author of the passage is primarily concerned with(内容性主题题)
A.        criticizing Locke’s economic theories
B.        discounting the contribution of labor in a modern economy
C.        questioning the validity of the labor theory of value
D.        arguing for a more equitable distribution of business profits
E.        contending that employers are overcompensated for capital goods

9. Which of the following arguments would a proponent of the labor theory of value, as it is presented in the first paragraph, be most likely to use in response to the statement that “The labor theory of value systematically disregards the productive contribution of capital goods” (lines 44-47)? (信息题)
A.        The productive contributions of workers and capital goods cannot be compared because the productive life span of capital goods is longer than that of workers.
B.        The author’s analysis of the distribution of income is misleading because only a small percentage of workers are also shareholders.
C.        Capital goods are valuable only insofar as they contribute directly to the production of consumer goods.
D.        The productive contribution of capital goods must be discounted because capital goods require maintenance.
E.        The productive contribution of capital goods must be attributed to labor because capital goods are themselves products of labor.
作者: s    时间: 2010-11-22 21:33

GWD-TN-15 Passage 2
题材类型:社会科学•美国妇女
结构类型:新老观点型
组织方式:总分,旧观点=>新观点=>解释
主要对象:resistance to patriarchal marriage
代表人物:Wulf
作者态度:大负小正
In her account of unmarried women’s experiences in colonial Philadelphia, Wulf argues that educated young women, particularly Quakers, engaged in resistance to patriarchal marriage by exchanging poetry critical of marriage, copying verse into their commonplace books. Wulf suggests that this critique circulated beyond the daughters of the Quaker elite and middle class, whose commonplace books she mines, proposing that Quaker schools brought it to many poor female students of diverse backgrounds.
Here Wulf probably overstates Quaker schools’ impact. At least three years’ study would be necessary to achieve the literacy competence necessary to grapple with the material she analyzes. In 1765, the year Wulf uses to demonstrate the diversity of Philadelphia’s Quaker schools, 128 students enrolled in these schools. Refining Wulf’s numbers by the information she provides on religious affiliation, gender, and length of study, it appears that only about 17 poor non-Quaker girls were educated in Philadelphia’s Quaker schools for three years or longer. While Wulf is correct that a critique of patriarchal marriage circulated broadly, Quaker schools probably cannot be credited with instilling these ideas in the lower classes. Popular literary satires on marriage had already landed on fertile ground in a multiethnic population that embodied a wide range of marital beliefs and practices. These ethnic- and class-based traditions themselves challenged the legitimacy of patriarchal marriage.

15. The primary purpose of the passage is to(内容性主题题)
A. argue against one aspect of Wulf’s account of how ideas critical of marriage were disseminated among young women in colonial Philadelphia
B. discuss Wulf’s interpretation of the significance for educated young women in colonial Philadelphia of the poetry they copied into their commonplace books
C. counter Wulf’s assertions about the impact of the multiethnic character of colonial Philadelphia’s population on the prevalent views about marriage
D. present data to undermine Wulf’s assessment of the diversity of the student body in Quaker schools in colonial Philadelphia
E. challenge Wulf’s conclusion that a critique of marriage was prevalent among young women of all social classes in colonial Philadelphia

16. According to the passage, which of the following was true of attitudes toward marriage in colonial Philadelphia? (信息题,可定位)
A. Exemplars of a critique of marriage could be found in various literary forms, but they did not impact public attitudes except among educated young women.
B. The diversity of the student body in the Quaker schools meant that attitudes toward marriage were more disparate there than elsewhere in Philadelphia society.
C. Although critical attitudes toward marriage were widespread, Quaker schools’ influence in disseminating these attitudes was limited.
D. Criticisms of marriage in colonial Philadelphia were directed at only certain limited aspects of patriarchal marriage.
E. The influence of the wide range of marital beliefs and practices present in Philadelphia’s multiethnic population can be detected in the poetry that educated young women copied in their commonplace books.

17. The author of the passage implies which of the following about the poetry mentioned in the first paragraph? (信息题,可定位,作者观点,在第二段)
A. Wulf exaggerates the degree to which young women from an elite background regarded the poetry as providing a critique of marriage.
B. The circulation of the poetry was confined to young Quaker women.
C. Young women copied the poetry into their commonplace books because they interpreted it as providing a desirable model of unmarried life.
D. The poetry’s capacity to influence popular attitudes was restricted by the degree of literacy necessary to comprehend it.
E. The poetry celebrated marital beliefs and practices that were in opposition to patriarchal marriage.

18. Which of the following, if true, would most seriously undermine the author’s basis for saying that Wulf overstates Quaker schools’ impact (lines 17-18)?(逻辑题)
A. The information that Wulf herself provided on religious affiliation and gender of students is in fact accurate.
B. Most poor, non-Quaker students enrolled in Quaker schools had completed one or two years’ formal or informal schooling before enrolling.
C. Not all of the young women whose commonplace books contained copies of poetry critical of marriage were Quakers.
D. The poetry featured in young women’s commonplace books frequently included allusions that were unlikely to be accessible to someone with only three years’ study in school.
E. In 1765 an unusually large proportion of the Quaker schools’ student body consisted of poor girls from non-Quaker backgrounds.
作者: s    时间: 2010-11-22 21:33

GWD-TN-15 Passage 3
题材类型:经济管理
结构类型:结论说明型
组织方式:总分,结论=>解释
主要对象:investment
代表人物:
作者态度:大负小正
    To compete effectively in international markets, a nation’s businesses must sustain investment in intangible as well as physical assets.  Although an enormous pool of investment capital exists in the United States, the country’s capital investment practices put United States companies at a competitive disadvantage.
    United States capital investment practices, shaped by sporadic and unpredictable changes in tax policy and high federal budget deficits, encourage both underinvestment and overinvestment. For example, United States companies invest at a low rate in internal development projects, such as improving supplier relations, that do not offer immediate profit, and systematically invest at a high rate in external projects, such as corporate takeovers, that yield immediate profit. Also, United States companies make too few linkages among different forms of investments. Such linkages are important because physical assets, such as factories, may not reach their potential level of productivity unless companies make parallel investments in intangible assets such as employee training and product redesign. In general, unlike Japanese and German investment practices, which focus on companies’ long-term interests, United States investment practices favor those forms of investment for which financial returns are most readily available. By making minimal investments in intangible assets, United States companies reduce their chances for future competitiveness.

23. The passage is primarily concerned with(内容性主题题)
A.        evaluating strategies for improving United States competitiveness in international markets
B.        illustrating the possible uses of investment capital
C.        analyzing some failings of capital investment practices in the United States
D.        suggesting reasons for increasing linkages among different types of investments
E.        contrasting the benefits of investment in physical assets with the benefits of investment in intangible assets

24. According to the passage, which of the following characterizes the capital allocation strategy of United States companies? (事实题)
A.        They tend to underinvest in intangible assets.
B.        They tend to invest heavily in product redesign.
C.        They tend to underinvest in physical assets.
D.        They make parallel investments in internal and external projects.
E.        They seek to allocate capital in ways that reduce their tax burden.

25. Which of the following best describes the purpose of the second paragraph? (举例题)
A.        To propose a solution to the problem introduced in the first paragraph
B.        To provide support for an argument presented in the first paragraph
C.        To provide data to refute an assertion made in the first paragraph
D.        To discuss the sources of investment capital mentioned in the first paragraph
E.        To discuss the competitiveness of international markets alluded to in the first paragraph
作者: s    时间: 2010-11-22 21:34

GWD-TN-15 Passage 4
题材类型:经济管理
结构类型:结论说明型
组织方式:总分,观点=>传统观点=>分析说明
主要对象:organization’s values
代表人物:
作者态度:大正小负
A pressing need in the study of organizations is for more research into how an organization’s values (an organization’s guiding principles and beliefs as perceived by its members) affect managerial decision-making. Traditional theories have been based on a “rational model,” which focuses on the decision-maker and either ignores the organizational value climate or conveniently assumes that the organization’s values are consistent or clearly prioritized. In reality, however, decisions are shaped not only by a manager’s own values, but also by those of the corporate culture and of organizational superiors. A recent study found that managers’ most stressful decisions involved “value contention” (conflicts among any of these sets of values). Furthermore, different types of organizational value systems were associated with different frequencies of contending values as well as with different types of managerial response. Explicit corporate values, for example, produced a greater percentage of decisions that were stressful due to value contention. Hidden values (those that an organization practices but does not acknowledge or which a superior furtively pursues in opposition to the values of the organization) produced a lower level of value contention. Although explicit values created more value contention, they were nonetheless more likely to produce flexible, well-reasoned decisions. Conversely, managers perplexed by hidden values reported feeling unable to identify an appropriate range of options.

35. The passage suggests that which of the following has resulted from the influence of the rational model (line 8) ? (信息题)
A.        It has deflected researchers’ attention from a critical factor affecting managerial decision-making.
B.        It has focused decision-making procedures on managers’ presumed ability to prioritize key corporate values.
C.        It has diverted attention from the need for orientation of nonsupervisory employees to organizational values.
D.        It has hampered communication between academic researchers and mangers of organizations.
E.        It has produced theories that are practicable for analyzing decision-making processes only in relatively large organizations.

36. The passage identifies which of the following as a way in which hidden corporate values affect managerial decision-making? (事实题)
A.        They tend to discourage consultation with organizational subordinates and superiors.
B.        They tend to undermine managers’ confidence in their own ability to determine the available alternatives.
C.        They tend to produce a heightened degree of value contention.
D.        They tend to produce a heightened degree of conflict among different levels of the organizational structure.
E.        They tend to cause greater anxiety among managers than do explicit corporate values.

37. According to the passage, value contention has been shown to affect managers by(事实题)
A.        decreasing their ability to conform to the values of the organization
B.        decreasing their ability to discern clearly the guiding principles and beliefs of the organization
C.        narrowing their range of options in the decision-making process
D.        increasing the frequency and intensity of conflicts with superiors and subordinates
E.        increasing the level of mental or emotional strain that accompanies the decision-making process
作者: s    时间: 2010-11-22 21:35

GWD-TN-16 Passage 1
题材类型:社会科学•美国女人
结构类型:新老观点型
组织方式:总分,旧观点=>分析=>新观点
主要对象:age and gender
代表人物:Robertson
作者态度:大正小负
    In a 1984 book, Claire C, Robertson argued that, before colonialism, age was a more important indicator of status and authority than gender in Ghana and in Africa generally. British colonialism imposed European- style male dominant notions upon more egalitarian local situations to the detriment of women generally, and gender became a defining characteristic that weakened women’s power and authority.
    Subsequent research in Kenya convinced Robertson that she had overgeneralized about Africa. Before colonialism, gender was more salient in central Kenya than it was in Ghana, although age was still crucial in determining authority. In contrast with Ghana, where women had traded for hundreds of years and achieved legal majority (not unrelated phenomena), the evidence regarding central Kenya indicated that women were legal minors and were sometimes treated as male property, as were European women at that time. Factors like strong patrilinearity and patrilocality, as well as women’s inferior land rights and lesser involvement in trade, made women more dependent on men than was generally the case in Ghana. However, since age apparently remained the overriding principle of social organization in central Kenya, some senior women had much authority. Thus, Robertson revised her hypothesis somewhat, arguing that in determining authority in precolonial Africa age was a primary principle that superseded gender to varying degrees depending on the situation.

6. The primary purpose of the passage is to(内容性主题题)
A.        present evidence undermining a certain hypothesis
B.        describe a particular position and its subsequent modification
C.        discuss two contrasting viewpoints regarding a particular issue
D.        describe how a social phenomenon varied by region
E.        evaluate an assumption widely held by scholars

7. The passage indicates that Robertson’s research in Kenya caused her to change her mind regarding which of the following? (信息题)
A.        Whether age was the prevailing principle of social organization in Kenya before colonialism
B.        Whether gender was the primary determinant of social authority in Africa generally before colonialism
C.        Whether it was only after colonialism that gender became a significant determinant of authority in Kenyan society
D.        Whether age was a crucial factor determining authority in Africa after colonialism
E.        Whether British colonialism imposed European-style male-dominant notions upon local situations in Ghana

8. The passage suggests that after conducting the research mentioned in line 18, but not before, Robertson would have agreed with which of the following about women’s status and authority in Ghana? (信息题)
A.        Greater land rights and greater involvement in trade made women in precolonial Ghana less dependent on men than were European women at that time.
B.        Colonialism had a greater impact on the status and authority of Ghanaian women than on Kenyan women.
C.        Colonialism had less of an impact on the status and authority of Ghanaian women that it had on the status and authority of other African women.
D.        The relative independence of Ghanaian women prior to colonialism was unique in Africa.
E.        Before colonialism, the status and authority of Ghanaian women was similar to that of Kenyan women.

9. The author of the passage mentions the status of age as a principle of social organization in precolonial central Kenya in lines 24-26 most likely in order to(举例题)
A.        indicate that women’s dependence on men in precolonial Kenya was not absolute
B.        contrast the situation of senior women to that of less senior women in precolonial Kenyan society
C.        differentiate between the status and authority of precolonial Kenyan women and that of precolonial Ghanaian women
D.        explain why age superseded gender to a greater extent in precolonial Kenya than it did elsewhere in Africa
E.        identify a factor that led Robertson to revise her hypothesis about precolonial Africa
作者: s    时间: 2010-11-22 21:37

GWD-TN-16 Passage 2
题材类型:自然科学•生命科学
结构类型:现象解释型
组织方式:总分,观点=>现象=>分析=>实验=>解释
主要对象:songbird
代表人物:
作者态度:大负小正
   Grassland songbirds often nest in the same grassland-wetland complexes as waterfowl, particularly in a certain part of those complexes, namely, upland habitats surrounding wetlands. Although some wildlife management procedures directed at waterfowl, such as habitat enhancement or restoration, may also benefit songbirds, the impact of others, especially the control of waterfowl predators, remains difficult to predict. For example, most predators of waterfowl nests prey opportunistically on songbird nests, and removing these predators could directly increase songbird nesting success. Alternatively, small mammals such as mice and ground squirrels are important in the diet of many waterfowl-nest predators and can themselves be important predators of songbird nets. Thus, removing waterfowl-nest predators could affect songbird nesting success through subsequent increases in small-mammal populations.
   In 1995 and 1996, researchers trapped and removed certain waterfowl-nest predators, primary raccoons and striped skunks, then observed subsequent survival rates for songbird nests. Surprisingly, they observed no significant effect on songbird nesting success. This may be due to several factors. Neither raccoons nor striped skunks consume ground squirrels, which are important predators of songbird nests. Thus, their removal may not have led to significant increases in populations of smaller predators. Additionally, both raccoons and striped skunks prefer wetlands and spend little time in upland habitats; removing these species may not have increased the nesting success of songbirds in the uplands enough to allow detection.

13. According to the passage, which of the following is true about the role played by ground squirrels in the ecology of grassland-wetland complexes? (事实题)
A.        While not important in the diet of raccoons or striped skunks, ground squirrels are a significant source of food for other waterfowl-nest predators.
B.        Whereas ground squirrels are typically important as predators of songbird nests, their opportunistic predation on waterfowl nests also has an observable effect on waterfowl nesting success.
C.        Although most waterfowl-nest predators prey on small mammals such as mice and ground squirrels, populations of ground squirrels tend to increase quickly enough to compensate for this level of predation.
D.        Although ground squirrels have been known to prey on songbird nests, a larger portion of their diets is usually provided by predation on waterfowl nests.
E.        Since larger predators tend to prefer small mammals to songbird eggs as a food source, a large population of ground squirrels plays an important role in controlling opportunistic predation on songbird nests.

14. Which of the following best describes the function of the sentence “Neither raccoons…songbird nests” (lines 34-37) in the context of the passage as a whole? (举例题)
A.        It raises questions about the validity of a theory described in the first paragraph.
B.        It points out an oversimplification that is inherent in the argument presented in the first paragraph.
C.        It introduces information that may help explain the results of the experiment that are presented earlier in the paragraph.
D.        It provides a specific example of the type of data collected in the experiment described earlier in the paragraph.
E.        It anticipates a potential objection to the conclusions drawn by the researchers involved in the experiment described earlier in the paragraph.

15. The primary purpose of the passage is to(内容性主题题)
A.        describe some procedures used for wildlife management and consider some problems associated with the execution of those procedures
B.        outline a problem related to a wildlife management procedure and offer potential explanations for the results of an experiment bearing on that problem
C.        present experimental results that illustrate the need for certain wildlife management procedures and point out some inconsistencies in those results
D.        argue that a certain procedure used for wildlife management should be modified because of its unintended consequences
E.        propose that further experiments be performed to assess the long-term effects of certain wildlife management procedures

16. The passage suggests that removing waterfowl-nest predators could possibly have a negative effect on songbird populations because(事实题)
A.        songbird populations could then grow to unsustainable numbers
B.        small-mammal population could then move out of the uplands into wetland areas
C.        competition among remaining waterfowl-nest predators could decrease significantly
D.        a resulting increase in waterfowl populations could crowd out songbird populations
E.        a resulting increase in small-mammal populations could increase small-mammal predation on songbirds

17. It can be inferred that the habitat preferences of raccoons and striped skunks affected the results of the experiment described in the passage for which of the following reasons? (事实题)
A.        Songbird nests in the wetlands are usually located in places that most waterfowl-nest predators cannot reach.
B.        Raccoons and striped skunks are not usually found in areas where songbird nests tend to be located.
C.        Mice and ground squirrels tend to avoid predation by raccoons and striped skunks by remaining exclusively in the uplands.
D.        The populations of small mammals in the wetlands are usually controlled by larger waterfowl-nest predators such as raccoons and striped skunks.
E.        The waterfowl on which raccoons and striped skunks prey in the wetlands compete with songbirds for food.
作者: s    时间: 2010-11-22 21:38

GWD-TN-16 Passage 3
题材类型:社会科学•美国女人
结构类型:现象解释型
组织方式:总分,现象=>分析=>解释
主要对象:women’s movement
代表人物:Socialist
作者态度:大正小负
    In mid-February 1917 a women’s movement independent of political affiliation erupted in New York City, the stronghold of the Socialist party in the United states. Protesting against the high cost of living, thousands of women refused to buy chickens, fish, and vegetables. The boycott shut down much of the City’s foodstuffs marketing for two weeks, riveting public attention on the issue of food prices, which had increased partly as a result of increased exports of food to Europe that had been occurring since the outbreak of the First World War.
    By early 1917 the Socialist party had established itself as a major political presence in New York City. New York Socialists, whose customary spheres of struggle were electoral work and trade union organizing, seized the opportunity and quickly organized an extensive series of cost-of-living protests designed to direct the women’s movement toward Socialist goals. Underneath the Socialists’ brief commitment to cost-of-living organizing lay a basic indifference to the issue itself. While some Socialists did view price protests as a direct step toward socialism, most Socialists ultimately sought to divert the cost-of-living movement into alternative channels of protest. Union organizing, they argued, was the best method through which to combat the high cost of living. For others, cost-of-living or organizing was valuable insofar as it led women into the struggle for suffrage, and similarly, the suffrage struggle was valuable insofar as it moved United States society one step closer to socialism.
    Although New York’s Socialists saw the cost-of-living issue as, at best, secondary or tertiary to the real task at hand, the boycotters, by sharp contrast, joined the price protest movement out of an urgent and deeply felt commitment to the cost-of-living issue. A shared experience of swiftly declining living standards caused by rising food prices drove these women to protest. Consumer organizing spoke directly to their daily lives and concerns; they saw cheaper food as a valuable end in itself. Food price protests were these women’s way of organizing at their own workplace, as workers whose occupation was shopping and preparing food for their families.

25. The author suggests which of the following about the New York Socialists’ commitment to the cost-of-living movement? (信息题)
A.        It lasted for a relatively short period of time.
B.        It was stronger than their commitment to the suffrage struggle.
C.        It predated the cost-of-living protests that erupted in 1917.
D.        It coincided with their attempts to bring more women into union organizing.
E.        It explained the popularity of the socialist party in New York City.

26. It can be inferred from the passage that the goal of the boycotting women was the(信息题)
A.        achievement of an immediate economic outcome
B.        development of a more socialistic society
C.        concentration of widespread consumer protests on the more narrow issue of food prices
D.        development of one among a number of different approaches that the women wished to employ in combating the high cost of living.
E.        attraction of more public interest to issues that the women and the New York socialists considered important.

27. Which of the following best states the function of the passage as a whole? (内容性主题题)
A.        To contrast the views held by the Socialist party and by the boycotting women of New York City on the cost-of-living issue
B.        To analyze the assumptions underlying opposing viewpoints within the New York socialist party of 1917
C.        To provide a historical perspective on different approaches to the resolution of the cost-of-living issue.
D.        To chronicle the sequence of events that led to the New York Socialist party’s emergence as a political power
E.        To analyze the motivations behind the socialist party’s involvement in the women’s suffrage movement.

28. According to the passage, most New York socialists believed which of the following about the cost-of-living movement? (信息题)
A.        It was primarily a way to interest women in joining the socialist party.
B.        It was an expedient that was useful only insofar as it furthered other goals.
C.        It would indirectly result in an increase in the number of women who belonged to labor unions.
D.        It required a long-term commitment but inevitably represented a direct step toward socialism.
E.        It served as an effective complement to union organizing.
作者: s    时间: 2010-11-22 21:38

GWD-TN-16 Passage 4
题材类型:经济管理
结构类型:现象解释型
组织方式:总分,现象=>分析=>解释
主要对象:acquisitions
代表人物:
作者态度:大负小正
    Findings from several studies on corporate mergers and acquisitions during the 1970’s and 1980’s raise questions about why firms initiate and consummate such transactions. One study showed, for example, that acquiring firms were on average unable to maintain acquired firms’ pre-merger levels of profitability. A second study concluded that post-acquisition gains to most acquiring firms were not adequate to cover the premiums paid to obtain acquired firms. A third demonstrated that, following the announcement of a prospective merger, the stock of the prospective acquiring firm tends to increase in value much less than does that of the firm for which it bids. Yet mergers and acquisitions remain common, and bidders continue to assert that their objectives are economic ones. Acquisitions may well have the desirable effect of channeling a nation’s resources efficiently from less to more efficient sectors of its economy, but the individual acquisitions executives arranging these deals must see them as advancing either their own or their companies’ private economic interests. It seems that factors having little to do with corporate economic interests explain acquisitions. These factors may include the incentive compensation of executives, lack of monitoring by boards of directors, and managerial error in estimating the value of firms targeted for acquisition. Alternatively, the acquisition acts of bidders may derive from modeling: a manager does what other managers do.

33. According to the passage, during the 1970’s and 1980’s bidding firms differed from the firms for which they bid in that bidding firms(事实题)
A.        tended to be more profitable before a merger than after a merger
B.        were more often concerned about the impact of acquisitions on national economies
C.        were run by managers whose actions were modeled on those of other managers
D.        anticipated greater economic advantages from prospective mergers
E.        experienced less of an increase in stock value when a prospective merger was announced

34. It can inferred from the passage that the author would be most likely to agree with which of the following statements about corporate acquisitions? (信息题)
A.        Their known benefits to national economies explain their appeal to individual firms during the 1970’s and 1980’s.
B.        Despite their adverse impact on some firms, they are the best way to channel resources from less to more productive sectors of a nation’s economy.
C.        They are as likely to occur because of poor monitoring by boards of directors as to be caused by incentive compensation for managers.
D.        They will be less prevalent in the future, since their actual effects will gain wider recognition.
E.        Factors other than economic benefit to the acquiring firm help to explain the frequency with which they occur.

35. The author of the passage implies that which of the following is a possible partial explanation for acquisition behavior during the 1970’s and 1980’s? (事实题)
A.        Managers wished to imitate other managers primarily because they saw how financially beneficial other firms’ acquisitions were.
B.        Managers miscalculated the value of firms that were to be acquired.
C.        Lack of consensus within boards of directors resulted in their imposing conflicting goals on managers.
D.        Total compensation packages for managers increased during that period.
E.        The value of bidding firms’ stock increased significantly when prospective mergers were announced.
作者: s    时间: 2010-11-22 21:39

GWD-TN-17 Passage 1
题材类型:自然科学•生命科学
结构类型:结论说明型
组织方式:总分,结论=>解释说明
主要对象:pheromone
代表人物:
作者态度:大正小负
There is no consensus among researchers regarding what qualifies a substance as a pheromone. While most agree on a basic definition of pheromones as chemicals released by one individual of a species which, when detected by another individual of the same species, elicit a specific behavioral or physiological response, some researchers also specify that the response to pheromones must be unconscious. In addition, the distinction between pheromones and odorants—chemicals that are consciously detected as odors—can be blurry, and some researchers classify pheromones as a type of odorant. Evidence that pheromone responses may not involve conscious odor perception comes from the finding that in many species, pheromones are processed by the vomeronasal (or accessory olfactory) system, which uses a special structure in the nose, the vomeronasal organ (VNO), to receive chemical signals. The neural connections between the VNO and the brain are separate from those of the main olfactory system, whose processing of odorants triggers sensations of smell. But while the VNO does process many animal pheromone signals, not all animal pheromones work through the VNO. Conversely, not all chemical signals transmitted via the VNO quality as pheromones. For example, garter snakes detect a chemical signal from earthworms—one of their favorite foods—via the VNO, and they use this signal to track their prey.

7. It can be inferred from the passage that in classifying pheromones as a type of odorant, the researchers referred to in line 15 posit that(事实题)
A.        pheromones are perceived consciously
B.        most pheromones are processed by the VNO
C.        most chemical signals processed by the VNO are pheromones
D.        pheromone perception does not occur exclusively between members of the same species.
E.        pheromones do not always elicit a specific behavioral or physiological response

8. According to the passage, the fact that pheromones are processed by the VNO in many animal species has been taken as evidence of which of the following? (举例题)
A.        The accessory and main olfacstory systems are not separate
B.        Odorants and pheromones are not distinct types of chemicals.
C.        Odorants and pheromones both elicit a specific behavioral response.
D.        Pheromones do not trigger conscious sensations of smell.
E.        Pheromones aid animals in tracking prey.

9. The primary purpose of the passage is to(内容性主题题)
A.        compare and contrast the ways in which the vomeronasal organ and the main olfactory systern process chemicals.
B.        summarize the debate over the role the vomeronasal organ plays in odor perception
C.        present some of the issues involved in the debate over what constitutes a pheromone
D.        propose a new definition of pheromones based on recent research
E.        argue that pheromones should be classified as a type of odorant
作者: s    时间: 2010-11-22 21:39

GWD-TN-17 Passage 2
题材类型:经济管理
结构类型:问题解决型
组织方式:总分,问题=>分析=>解决方案
主要对象:board performance
代表人物:
作者态度:大负小正
    Although recent censure of corporate boards of directors as “passive” and “supine” may be excessive, those who criticize board performance have plenty of substantive ammunition. Too many corporate boards fail in their two crucial responsibilities of overseeing long-term company strategy and of selecting, evaluating, and determining appropriate compensation of top management. At times, despite disappointing corporate performance, compensation of chief executive officers reaches indefensibly high levels, nevertheless, suggestions that the government should legislate board reform are premature. There are ample opportunities for boards themselves to improve corporate performance.
    Most corporate boards’ compensation committees focus primarily on peer-group comparisons. They are content if the pay of top executives approximates that of the executives of competing firms with comparable short-term earnings or even that of executives of competing firms of comparable size. However, mimicking the compensation policy of competitors for the sake of parity means neglecting the value of compensation as a means of stressing long-term performance. By tacitly detaching executive compensation policy from long-term performance, committees harm their companies and the economy as a whole. The committees must develop incentive compensation policies to emphasize long-term performance. For example a board’s compensation committee can, by carefully proportioning straight salary and such short-term and long-term incentives as stock options, encourage top management to pursue a responsible strategy.

14. According to the passage, the majority of compensation committees put the greatest emphasis on which of the following when determining compensation for their executives? (事实题)
A.        Long-term corporate performance
B.        The threat of government regulation
C.        Salaries paid to executives of comparable corporations
D.        The probable effect the determination will have on competitors
E.        The probable effect the economic climate will have on the company

15. The passage suggests which of the following about government legislation requiring that corporate boards undergo reform? (事实题)
A.        Such legislation is likely to discourage candidates from joining corporate boards.
B.        Such legislation is likely to lead to reduced competition among companies.
C.        The performance of individual companies would be affected by such legislation to a greater extent than would the economy as a whole.
D.        Such legislation would duplicate initiatives already being made by corporate boards to improve their own performance.
E.        Corporate boards themselves could act to make such legislation unnecessary.

16. Which of the following best describes the organization of the passage? (写法性主题题)
A.        A problem is acknowledged, the causes are explored, and a solution is offered.
B.        A question is raised, opposing points of view are evaluated, and several alternative answers are discussed.
C.        A means of dealing with a problem is proposed, and the manner in which a solution was reached is explained.
D.        A plan of action is advanced, and the probable outcomes of that plan are discussed.
E.        Two competing theories are described and then reconciled.
作者: s    时间: 2010-11-22 21:40

GWD-TN-17 Passage 3
题材类型:社会科学•美国女人
结构类型:现象解释型
组织方式:总分,现象=>分析=>解释
主要对象:women in civil cases
代表人物:Dayton
作者态度:大负小正
    In colonial Connecticut between 1670 and 1719, women participated in one of every six civil cases, the vast majority of which were debt-related. Women’s participation dropped to one in ten cases after 1719, and to one in twenty by the 1770’s. However, as Cornelia Hughes Dayton notes in Women Before the Bar: Gender, Law, and Society in Connecticut, 1639-1789, these statistics are somewhat deceptive: in fact, both the absolute numbers and the percentage of adult women participating in civil cases grew steadily throughout the eighteenth century, but the legal activity of men also increased dramatically, and at a much faster rate. Single, married, and widowed women continued to pursue their own and their husbands’ debtors through legal action much as they had done in the previous century, but despite this continuity, their place in the legal system shifted dramatically. Men’s commercial interests and credit networks became increasingly far-flung, owing in part to the ability of creditors to buy and sell promissory notes (legal promises to pay debts). At the same time, women’s networks of credit and debt remained primarily local and personal. Dayton contends that, although still performing crucial economic services in their communities—services that contributed to the commercialization of the colonial economy—women remained for the most part outside the new economic and legal culture of the eighteenth century.

25. The passage is primarily concerned with(写法性主题题)
A.        reporting an author’s view of a phenomenon
B.        disputing the reasons usually given for an unexpected change
C.        evaluating the conclusions reached by an author
D.        assessing the impact of certain legal decisions.
E.        defending a controversial point of view

26. According to the passage, compared with women in eighteenth-century Connecticut, men were(取非题)
A.        more likely to rely on credit and go into debt
B.        more likely to pursue their families’ debtors
C.        more likely to participate in economic transactions outside their own communities
D.        less likely to perform economic services in their own communities.
E.        less likely to participate in civil cases that were not debt-related.

27. The passage suggests that which of the following best compares the economic concerns of women with those of men toward the close of the eighteenth Century in colonial Connecticut? (信息题)
A.        Both men and women had more economic responsibilities at the end of the century than they had had at the beginning of the century.
B.        Women’s economic activities had become less varied by the end of the century; men’s economic activities had become more varied.
C.        Women’s economic activities at the end of the century were similar to their activities at the beginning; men’s economic activities changed considerably.
D.        Women’s economic concerns at the end of the century were primarily familial; men’s economic concerns were primarily political.
E.        Women’s economic concerns at the end of the century were primarily personal; men’s economic needs were primarily familial.
作者: s    时间: 2010-11-22 21:40

GWD-TN-17 Passage 4
题材类型:自然科学
结构类型:新老观点型
组织方式:总分,旧方法=>新方法=>新方法说明
主要对象:measure Mount Everest
代表人物:Chinese
作者态度:负评价
    In 1975 Chinese survey teams remeasured Mount Everest, the highest of the Himalayan mountains. Like the British in 1852, they used the age-old technique of “carrying in” sea level: surveyors marched inland from the coast for thousands of miles, stopping at increments of as little as a few feet to measure their elevation, and marking each increment with two poles. To measure the difference in elevation between poles, surveyors used an optical level—a telescope on a level base—placed halfway between the poles. They sighted each pole, reading off measurements that were then used to calculate the change in elevation over each increment. In sight of the peaks the used theodolites telescopes for measuring vertical and horizontal angles—to determine the elevation of the summit.
    The Chinese, however, made efforts to correct for the errors that had plagued the British. One source of error is refraction, the bending of light beams as they pass through air layers of different temperature and pressure. Because light traveling down from a summit passes through many such layers, a surveyor could sight a mirage rather than the peak itself. To reduce refraction errors, the Chinese team carried in sea level to within five to twelve miles of Everest’s summit, decreasing the amount of air that light passed through on its way to their theodolites. The Chinese also launched weather balloons near their theodolites to measure atmospheric temperature and pressure changes to better estimate refraction errors. Another hurdle is the peak’s shape. When surveyors sight the summit, there is a risk they might not all measure the same point. In 1975 the Chinese installed the first survey beacon on Everest, a red reflector visible through a theodolite for ten miles, as a reference point. One more source of error is the unevenness of sea level. The British assumed that carrying in sea level would extend an imaginary line from the shore along Earth’s curve to a point beneath the Himalaya. In reality, sea level varies according to the irregular interior of the planet. The Chinese used a gravity meter to correct for local deviations in sea level.

32. It can be inferred from the passage that refraction would be most likely to cause errors in measurements of a mountain’s elevation under which of the following conditions? (信息题)
A.        When there are local variations in sea level
B.        When light passes through humid air
C.        When theodolites are used relatively far from the mountain peak.
D.        When weather balloons indicate low air temperature and pressure.
E.        When sea level has been carried in to within five to twelve miles of the summit.

33. Which of the following best describes the purpose of the sentence in lines 23-25 (“The Chinese…the British”)? (举例题)
A.        Introduce a definition
B.        Signal a transition in focus
C.        Summarize the preceding paragraph
D.        Draw a contrast between two different theories.
E.        Present information that contradicts the preceding paragraph.

34. Which of the following is NOT mentioned as a possible source of error in surveying mountain elevation? (列举题)
A.        Mirages
B.        Refraction
C.        Inaccurate instruments
D.        Variations in sea level
E.        Uncertainty about the exact point to be measured

35. The primary purpose of the passage is to (写法性主题题)
A.        provide details about improvements to a process
B.        challenge the assumptions underlying a new method
C.        criticize the way in which a failed project was carried out
D.        call for new methods to solve an existing problem
E.        explain the theory behind a new technique
作者: s    时间: 2010-11-22 21:41

GWD-TN-17 Passage 4
题材类型:自然科学
结构类型:新老观点型
组织方式:总分,旧方法=>新方法=>新方法说明
主要对象:measure Mount Everest
代表人物:Chinese
作者态度:负评价
    In 1975 Chinese survey teams remeasured Mount Everest, the highest of the Himalayan mountains. Like the British in 1852, they used the age-old technique of “carrying in” sea level: surveyors marched inland from the coast for thousands of miles, stopping at increments of as little as a few feet to measure their elevation, and marking each increment with two poles. To measure the difference in elevation between poles, surveyors used an optical level—a telescope on a level base—placed halfway between the poles. They sighted each pole, reading off measurements that were then used to calculate the change in elevation over each increment. In sight of the peaks the used theodolites telescopes for measuring vertical and horizontal angles—to determine the elevation of the summit.
    The Chinese, however, made efforts to correct for the errors that had plagued the British. One source of error is refraction, the bending of light beams as they pass through air layers of different temperature and pressure. Because light traveling down from a summit passes through many such layers, a surveyor could sight a mirage rather than the peak itself. To reduce refraction errors, the Chinese team carried in sea level to within five to twelve miles of Everest’s summit, decreasing the amount of air that light passed through on its way to their theodolites. The Chinese also launched weather balloons near their theodolites to measure atmospheric temperature and pressure changes to better estimate refraction errors. Another hurdle is the peak’s shape. When surveyors sight the summit, there is a risk they might not all measure the same point. In 1975 the Chinese installed the first survey beacon on Everest, a red reflector visible through a theodolite for ten miles, as a reference point. One more source of error is the unevenness of sea level. The British assumed that carrying in sea level would extend an imaginary line from the shore along Earth’s curve to a point beneath the Himalaya. In reality, sea level varies according to the irregular interior of the planet. The Chinese used a gravity meter to correct for local deviations in sea level.

32. It can be inferred from the passage that refraction would be most likely to cause errors in measurements of a mountain’s elevation under which of the following conditions? (信息题)
A.        When there are local variations in sea level
B.        When light passes through humid air
C.        When theodolites are used relatively far from the mountain peak.
D.        When weather balloons indicate low air temperature and pressure.
E.        When sea level has been carried in to within five to twelve miles of the summit.

33. Which of the following best describes the purpose of the sentence in lines 23-25 (“The Chinese…the British”)? (举例题)
A.        Introduce a definition
B.        Signal a transition in focus
C.        Summarize the preceding paragraph
D.        Draw a contrast between two different theories.
E.        Present information that contradicts the preceding paragraph.

34. Which of the following is NOT mentioned as a possible source of error in surveying mountain elevation? (列举题)
A.        Mirages
B.        Refraction
C.        Inaccurate instruments
D.        Variations in sea level
E.        Uncertainty about the exact point to be measured

35. The primary purpose of the passage is to (写法性主题题)
A.        provide details about improvements to a process
B.        challenge the assumptions underlying a new method
C.        criticize the way in which a failed project was carried out
D.        call for new methods to solve an existing problem
E.        explain the theory behind a new technique
作者: 小宇宙2010    时间: 2010-11-23 06:31

many  thanks@
作者: citidandan    时间: 2010-11-30 06:34

many  thanks @@@
作者: cckjenius    时间: 2010-12-1 06:34

many  thanks@@
作者: snybest    时间: 2011-7-12 19:51

谢谢这么详细的分析!
作者: linlongbao    时间: 2011-7-17 11:17

~~~~~~~~~~~~~~~~~~~~~赞
作者: lojajaja    时间: 2012-6-29 10:26

THANKS
作者: lsaurora    时间: 2012-6-30 06:34

many thanks @
作者: zhuxiaocao    时间: 2012-7-3 06:25

many thanks A@
作者: torresAing    时间: 2012-7-7 10:57

many   thanks !!
作者: wltcharlotte    时间: 2012-7-9 20:18

谢谢!~
作者: viennawong    时间: 2012-7-26 11:19

many  thanks !!
作者: biglinda0806    时间: 2016-6-2 05:41

总结很到位,参考学些了




欢迎光临 国际顶尖MBA申请交流平台--TOPWAY MBA (http://forum.topway.org/) Powered by Discuz! 7.2